Assessment HEMA Flashcards

1
Q

One chromosome breaks off and becomes attached to a different chromosome:

Addition
Deletion
Inversion
Translocation

A

Translocation

How well did you know this?
1
Not at all
2
3
4
5
Perfectly
2
Q

CHROMOSOMAL CHANGES

Most common type of DNA change that can lead to leukemia.
A translocation means that a part of one chromosome breaks off and becomes attached to a different chromosome.
The point at which the break occurs can affect nearby genes—for example, it can turn on oncogenes or turn off genes that would normally help a cell to mature.

A

TRANSLOCATION

How well did you know this?
1
Not at all
2
3
4
5
Perfectly
3
Q

Occur when part of a chromosome is lost.
This may result in the cell losing a gene that helped keep its growth in check, for example, a tumor suppressor gene.

A

DELETION

How well did you know this?
1
Not at all
2
3
4
5
Perfectly
4
Q

Occur when part of a chromosome gets turned around, so it is now in reverse order.
This can result in the loss of a gene (or genes) because the cell can no longer read its instructions in protein translation.

A

INVERSION

How well did you know this?
1
Not at all
2
3
4
5
Perfectly
5
Q

An extra chromosome or part of a chromosome is gained.
This can lead to too many copies of certain genes within the cell.
This can be a problem if one or more of these genes are oncogenes.

A

ADDITION

How well did you know this?
1
Not at all
2
3
4
5
Perfectly
6
Q

The most versatile type of stem cell, can develop into any human cell type, including development from embryo into fetus:

Multipotential stem cell
Pluripotential stem cell
Totipotential stem cell

A

Totipotential stem cell

How well did you know this?
1
Not at all
2
3
4
5
Perfectly
7
Q

TYPES OF HUMAN STEM CELLS

These cells are present in the first few hours after an ovum is fertilized.
Totipotential stem cells, the most versatile type of stem cell, can develop into any human cell type, including development from embryo into fetus.

A

Totipotential stem cells

How well did you know this?
1
Not at all
2
3
4
5
Perfectly
8
Q

TYPES OF HUMAN STEM CELLS

These cells are present several days after fertilization.
Pluripotent stem cells can develop into any cell type, except they cannot develop into a fetus.

A

Pluripotential stem cells

How well did you know this?
1
Not at all
2
3
4
5
Perfectly
9
Q

TYPES OF HUMAN STEM CELLS

These cells are derived from pluripotent stem cells.
They can be found in adults, but they are limited to specific types of cells to form tissues.
For example, bone marrow stem cells can produce all types of blood cells, bone cartilage, and adipose (fat) cells.

A

Multipotential stem cells

How well did you know this?
1
Not at all
2
3
4
5
Perfectly
10
Q

The promyelocyte stage lasts about:

4 hours
12 hours
15 hours
24 hours
4.3 days

A

24 hours

How well did you know this?
1
Not at all
2
3
4
5
Perfectly
11
Q

PROLIFERATIVE PHASE

First identifiable cell in the granulocytic series
Constitute approximately 1% of the total nucleated bone marrow cells
Stage lasts approximately 15 hours

A

Myeloblast

How well did you know this?
1
Not at all
2
3
4
5
Perfectly
12
Q

Constitutes approximately 3% of the nucleated bone marrow cells
This stage lasts about 24 hours

A

Promyelocyte

How well did you know this?
1
Not at all
2
3
4
5
Perfectly
13
Q

Approximately 12% of the proliferative cells existing in this stage
Myelocyte to metamyelocyte lasts an average of 4.3 days.
Once the metamyelocyte stage has been reached, cells have undergone four or five cell divisions and the proliferative phase comes to an end

A

Myelocyte

How well did you know this?
1
Not at all
2
3
4
5
Perfectly
14
Q

MATURATION-STORAGE PHASE

A

Metamyelocytes 45%
Band 35%
Segmented granulocytes 20%

Segmented neutrophils in the maturation-storage compartment are frequently referred to as the marrow reserve.
This reserve constitutes a 4- to 8-day supply of neutrophils.

How well did you know this?
1
Not at all
2
3
4
5
Perfectly
15
Q

The tourniquet should be applied __________ inches above the venipuncture site.

1 to 2 inches above the venipuncture site
3 to 4 inches above the venipuncture site
5 to 6 inches above the venipuncture site
8 to 9 inches above the venipuncture site

A

3 to 4 inches above the venipuncture site

The tourniquet should be applied 3 to 4 inches above the venipuncture site and left on for no longer than 1 minute before the venipuncture is performed.

How well did you know this?
1
Not at all
2
3
4
5
Perfectly
16
Q

Anticoagulants that remove calcium needed for clotting by forming insoluble calcium salts: RODAK

EDTA
EDTA and heparin
EDTA and citrate
EDTA, citrate and oxalate
EDTA, citrate, oxalate and heparin

A

EDTA, citrate, and oxalate

Heparin prevents clotting by binding to anti- thrombin in the plasma and inhibiting
thrombin and activated coagulation factor X.

How well did you know this?
1
Not at all
2
3
4
5
Perfectly
17
Q

Number of inversions of light blue top evacuated tube:

None
3 to 4
5 to 6
8

A

3 to 4

Light blue (citrate) 3 to 4x inversions
Green (heparin) 8x inversions
Purple (EDTA) 8x inversions

How well did you know this?
1
Not at all
2
3
4
5
Perfectly
18
Q

Test orders: 1. Conduct continuous utilization reviews to ensure that physician laboratory orders are comprehensive and appropriate to patient condition; 2. Inform physician about laboratory test availability and ways to avoid unnecessary orders; 3. Reduce unnecessary repeat testing.

1 and 2
1 and 3
2 and 3
1, 2 and 3

A

1, 2 and 3

How well did you know this?
1
Not at all
2
3
4
5
Perfectly
19
Q

Each new assay or assay modification must be validated for: 1. Accuracy, Precision; 2. Linearity; 3. Specificity; 4. Lower limit of detection ability

1 and 3
2 and 4
1, 2 and 3
1, 2, 3 and 4

A

1, 2, 3 and 4

How well did you know this?
1
Not at all
2
3
4
5
Perfectly
20
Q

Adjuvant for infectious disease therapy:

Interleukin 2
Interleukin 3
Interleukin 6
Interleukin 12

A

Interleukin 12

How well did you know this?
1
Not at all
2
3
4
5
Perfectly
21
Q

Major elements of the flow cytometer: 1. Optics; 2. Fluidics; 3. Computer; 4. Electronics

1 and 2
3 and 4
1, 2 and 3
1, 2, 3 and 4

A

1, 2, 3 and 4

How well did you know this?
1
Not at all
2
3
4
5
Perfectly
22
Q

Total area of the Levy chamber with improved Neubauer ruling:

1 mm2
3 mm2
4 mm2
9 mm2

A

9 mm2

It is composed of two raised surfaces, each with a 3 mm x 3 mm square counting area or
grid (total area 9 mm2), separated by an H-shaped moat.

How well did you know this?
1
Not at all
2
3
4
5
Perfectly
23
Q

For the manual WBC count: After the chamber is filled, allow the cells to settle for___ minutes before counting.

3 minutes
5 minutes
10 minutes
15 minutes

A

10 minutes

How well did you know this?
1
Not at all
2
3
4
5
Perfectly
24
Q

Typical dilution for the manual platelet count:
1:10
1:20
1:100
1:200

A

1:100

How well did you know this?
1
Not at all
2
3
4
5
Perfectly
25
Q

If fewer than 50 platelets are counted on each side, the procedure should be repeated by diluting the blood to:

1:10
1:20
1:100
1:200

A

1:20

How well did you know this?
1
Not at all
2
3
4
5
Perfectly
26
Q

In the cyanmethemoglobin method, full conversion of hemoglobin to cyanmethemoglobin:

3 minutes
5 minutes
10 minutes
15 minutes

A

10 minutes

How well did you know this?
1
Not at all
2
3
4
5
Perfectly
27
Q

WBC count that can interfere with the cyanmethemoglobin method:

Greater than 4 x 10 9/L
Greater than 7 x 10 9/L
Greater than 11 x 10 9 /L
Greater than 20 x 10 9/L

A

Greater than 20 x 10 9/L

A high WBC count (greater than 20 x 10 9/L) or a high platelet count (greater than 700 x 10
9/L) can cause turbidity and a falsely high result.
In this case, the reagent-sample solution can be centrifuged and the supernatant
measured.

How well did you know this?
1
Not at all
2
3
4
5
Perfectly
28
Q

Effect of dehydration to hematocrit reading:

Decreased
Increased
Variable
No effect

A

Increased
## Footnote

The fluid loss associated with dehydration causes a decrease in plasma volume and
falsely increases the hematocrit reading.

How well did you know this?
1
Not at all
2
3
4
5
Perfectly
29
Q

An MCHC between 36 and 38 g/dL should be checked for:

Codocytes
Drepanocytes
Elliptocytes
Spherocytes

A

Spherocytes

How well did you know this?
1
Not at all
2
3
4
5
Perfectly
30
Q

In the manual reticulocyte count, what is the ratio of blood and new methylene blue stain?

1:1
1:2
1:3
1:4

A

1:1

Mix equal amounts of blood and new methylene blue stain (2 to 3 drops, or approximately
50 mL each), and allow to incubate at room temperature for 3 to 10 minutes.

How well did you know this?
1
Not at all
2
3
4
5
Perfectly
31
Q

To improve accuracy of the reticulocyte count, have another laboratorian count the other film; counts should agree within:

Within 1%
Within 5%
Within 10%
Within 20%

A

Within 20%

How well did you know this?
1
Not at all
2
3
4
5
Perfectly
32
Q

The ESR of patients with severe anemia is:

Critical
Of diagnostic significance
Of little diagnostic value

A

Of little diagnostic value

because it will be falsely elevated.

How well did you know this?
1
Not at all
2
3
4
5
Perfectly
33
Q

ESR and RBC mass:

Directly proportional
Inversely proportional
Cannot be determined

A

Directly proportional

The ESR is directly proportional to the red blood cell mass and inversely proportional to
plasma viscosity.

How well did you know this?
1
Not at all
2
3
4
5
Perfectly
34
Q

ESR of patient with leukemia:

Decreased
Increased
Variable

A

Increased

How well did you know this?
1
Not at all
2
3
4
5
Perfectly
35
Q

ESR of patient with leukocytosis:

Decreased
Increased
Variable

A

Decreased

How well did you know this?
1
Not at all
2
3
4
5
Perfectly
36
Q

An early indication of engraftment success after hematopoietic stem cell transplant. 1. RBC count; 2. Immature reticulocyte fraction; 3. Immature platelet fraction

2 only
1 and 2
2 and 3
1, 2 and 3

A

2 and 3

The immature reticulocyte fraction and the immature platelet fraction provide an early
indication of engraftment success after hematopoietic stem cell transplant.

How well did you know this?
1
Not at all
2
3
4
5
Perfectly
37
Q

Conditions associated with DIC: Examples of conditions associated with ENDOTOXINS THAT ACTIVATE CYTOKINES

Acute promyelocytic or myelomonocytic leukemia
Bacterial, protozoal, fungal and viral infections
Coronary artery bypass surgery
Hypovolemic and hemorrhagic shock

A

Bacterial, protozoal, fungal and viral infections

How well did you know this?
1
Not at all
2
3
4
5
Perfectly
38
Q

Fibrinogen concentration in primary fibrinolysis:

Decreased
Increased
Variable

A

Decreased

How well did you know this?
1
Not at all
2
3
4
5
Perfectly
39
Q

Fibrinogen concentration in secondary fibrinolysis:

Decreased
Increased
Variable

A

Decreased

How well did you know this?
1
Not at all
2
3
4
5
Perfectly
40
Q

Bone marrow reticulocytes have an average maturation of:

1 day
1.5 days
2 days
2.5 days

A

2.5 days

How well did you know this?
1
Not at all
2
3
4
5
Perfectly
41
Q

Once young reticulocytes enter the circulating blood, they remain in the reticulocyte stage for an average of:

1 day
1.5 days
2 days
2.5 days

A

1 day

Bone marrow reticulocytes have an average maturation period of 2.5 days.

Once young reticulocytes enter the circulating blood, they remain in the reticulocyte stage
for an average of 1 day and represent approximately 0.5% to 1.5% of the circulating
erythrocytes.

How well did you know this?
1
Not at all
2
3
4
5
Perfectly
42
Q

Basophils have an average circulation time of about:

7 to 10 hours
8.5 hours
12 hours
2.5 days

A

8.5 hours

Bone marrow reticulocytes have an average maturation period of 2.5 days.

Once young reticulocytes enter the circulating blood, they remain in the reticulocyte stage
for an average of 1 day and represent approximately 0.5% to 1.5% of the circulating
erythrocytes.

How well did you know this?
1
Not at all
2
3
4
5
Perfectly
43
Q

All identifiable patient information, whether written, computerized, visually, or audio recorded, or simply held in the memory of healthcare professionals, is subject to the duty of confidentiality, EXCEPT:

Any clinical information about an individual’s diagnosis or treatment
A picture, photograph, video, audiotape, or other images of the patient
The patient’s favorite restaurant and food
Who the patient’s doctor is and what clinics patients attend and when

A

The patient’s favorite restaurant and food

How well did you know this?
1
Not at all
2
3
4
5
Perfectly
44
Q

It is used by medical laboratories in developing their quality management systems and assessing their own competence and for use by accreditation bodies in confirming or recognizing the competence of medical laboratories:

ISO 11166
ISO 11469
ISO 15189
ISO 15819

A

ISO 15189

ISO 15189:2007 is for use by medical laboratories in developing their quality management
systems and assessing their own competence and for use by accreditation bodies in
confirming or recognizing the competence of medical laboratories.

How well did you know this?
1
Not at all
2
3
4
5
Perfectly
45
Q

All of the following are examples of pre-analytical errors, EXCEPT:

Specimen obtained from the wrong patient
Specimen collected in the wrong tube or container
Incorrect labeling of specimen
Failure to report critical values immediately

A

Failure to report critical values immediately

PREANALYTICAL (PREEXAMINATION)
■ Specimen obtained from the wrong patient
■ Specimen procured at the wrong time
■ Specimen collected in the wrong tube or container
■ Blood specimens collected in the wrong order
■ Incorrect labeling of specimen
■ Improper processing of specimen

ANALYTICAL (EXAMINATION)
■ Oversight of instrument flags
■ Out-of-control QC results
■ Wrong assay performed

POSTANALYTICAL (POSTEXAMINATION)
■ Verbal reporting of results
■ Instrument: Laboratory Information System (LIS) incompatibility error
■ Confusion about reference ranges
■ Failure to report critical values immediately

How well did you know this?
1
Not at all
2
3
4
5
Perfectly
46
Q

EDTA is used in concentrations of _____ of whole blood

0.5 mg/1 mL of whole blood
1 mg/1mL of whole blood
1.5 mg/1 mL of whole blood
2 mg/1 mL of whole blood

A

1.5 mg/1 mL of whole blood

How well did you know this?
1
Not at all
2
3
4
5
Perfectly
47
Q

Uncommon vascular complications that are not usually related to the technique include:

Pseudoaneurysm
Pseudoaneurysm and thrombosis
Pseudoaneurysm, thrombosis and reflex arteriospasm
Pseudoaneurysm, thrombosis, reflex arteriospasm and arteriovenous fistula formation

A

Pseudoaneurysm, thrombosis, reflex arteriospasm and arteriovenous fistula formation

How well did you know this?
1
Not at all
2
3
4
5
Perfectly
48
Q

Complications include orthostatic hypotension, syncope and shock

Vascular complications
Cardiovascular complications
Neurological complications
Dermatological comlications

A

Cardiovascular complications

How well did you know this?
1
Not at all
2
3
4
5
Perfectly
49
Q

Complications include diaphoresis, seizure and pain:

Vascular complications
Cardiovascular complications
Neurological complications
Dermatological comlications

A

Neurological complications

How well did you know this?
1
Not at all
2
3
4
5
Perfectly
50
Q

Sister chromatids move to the equatorial plate.

Prophase
Metaphase
Anaphase
Telophase

A

Metaphase

Characteristics of the Four Mitotic Periods

PROPHASE
The chromatin becomes tightly coiled.
Nucleolus and nuclear envelope disintegrate.
Centrioles move to opposite poles of the cell.

METAPHASE
Sister chromatids move to the equatorial plate.

ANAPHASE
Sister chromatids separate and move to opposite poles.

TELOPHASE
Chromosomes arrive at opposite poles.
Nucleolus and nuclear membrane reappear.
The chromatin pattern reappears.

How well did you know this?
1
Not at all
2
3
4
5
Perfectly
51
Q

Megakaryocytes develop into platelets in approximately __ days.

3 days
5 days
9 days
12 days

A

5 days

How well did you know this?
1
Not at all
2
3
4
5
Perfectly
52
Q

This cytokine promotes the growth of early hematopoietic cell lines:

Interleukin 1
Interleukin 2
Interleukin 3
Interleukin 6

A

Interleukin 3

Promotes the growth of early hematopoietic cell lines (e.g., proliferation of CFU-GEMM,
CFU-M, CFU-Meg, CFU-Eo, and CFU-Bs colonies from bone marrow).
IL-3 acts with M-CSF to stimulate proliferation of monocytes and macrophages. It also
stimulates granulocyte, monocyte, eosinophil, and mast cell production

How well did you know this?
1
Not at all
2
3
4
5
Perfectly
53
Q

Hemoglobin appears for the first time:

Rubriblast (pronormoblast)
Prorubricyte (basophilic normoblast)
Rubricyte (polychromatophilic normoblast)
Metarubricyte (orthochromic normoblast)

A

Rubricyte (polychromatophilic normoblast)

How well did you know this?
1
Not at all
2
3
4
5
Perfectly
54
Q

This pathway prevents denaturation of globin of the hemoglobin molecule by oxidation:

Embden-Meyerhof pathway
Hexose-monophosphate shunt
Methemoglobin reductase pathway
Luebering-Rapoport pathway

A

Hexose-monophosphate shunt

Embden-Meyerhof Pathway
Maintains cellular energy by generating ATP

Oxidative pathway or hexose-monophosphate shunt
Prevents denaturation of globin of the hemoglobin molecule by oxidation

Methemoglobin reductase pathway
Prevents oxidation of heme iron

Luebering-Rapaport pathway
Regulates oxygen affinity of hemoglobin

How well did you know this?
1
Not at all
2
3
4
5
Perfectly
55
Q

RBCs inclusions, 0.2 to 2.0 mm in size, that can be seen with a stain such as crystal violet or brilliant cresyl blue; represent precipitated, denatured hemoglobin and are clinically associated with congenital hemolytic anemia, G6PD deficiency, hemolytic anemias secondary to drugs such as phenacetin, and some hemoglobinopathies.

Hemoglobin C crystals
Heinz bodies
Howell-Jolly bodies
Pappenheimer bodies

A

Heinz bodies

How well did you know this?
1
Not at all
2
3
4
5
Perfectly
56
Q

RBC inclusions that are aggregates of mitochondria, ribosomes, and iron particles. Clinically, they are associated with iron-loading anemias, hyposplenism, and hemolytic anemias.

Basophilic stippling
Heinz bodies
Howell-Jolly bodies
Pappenheimer bodies

A

Pappenheimer bodies

How well did you know this?
1
Not at all
2
3
4
5
Perfectly
57
Q

RBC inclusions representing granules composed of ribosomes and RNA that are precipitated during the process of staining of a blood smear; associated clinically with disturbed erythropoiesis (defective or accelerated heme synthesis),lead poisoning, and severe anemias.

Basophilic stippling
Heinz bodies
Howell-Jolly bodies
Pappenheimer bodies

A

Basophilic stippling

How well did you know this?
1
Not at all
2
3
4
5
Perfectly
58
Q

Nuclear remnants predominantly composed of DNA; believed to develop in periods of accelerated or abnormal erythropoiesis, because the spleen cannot keep upwith pitting these remnants from the cell. Its presence is associated with hemolytic anemias, pernicious anemia, and particularly post-splenectomy, physiologicalatrophy of the spleen.

Basophilic stippling
Heinz bodies
Howell-Jolly bodies
Pappenheimer bodies

A

Howell-Jolly bodies

How well did you know this?
1
Not at all
2
3
4
5
Perfectly
59
Q

Anemias with low MCV and MCHC; microcytic, hypochromic RBCs, EXCEPT:

Iron deficiency anemia
Thalassemia
Sideroblastic anemia
Excessive alcohol ingestion

A

Excessive alcohol ingestion

Low MCV, MCHC
Microcytic, hypochromic
Typical of maturation defects
Iron deficiency anemia (some)
Thalassemia
Sideroblastic anemia

Normal MCV, MCHC
Normocytic, normochromic
Typical of hypoproliferation
Bone marrow disorder
Iron deficiency anemia (some)
Anemia of chronic disorders
Autoimmune disease

High MCV
Macrocytic
Typical of maturation defect
Vitamin B12 deficiency
Folate deficiency
Excessive alcohol ingestion
Hypothyroidism

How well did you know this?
1
Not at all
2
3
4
5
Perfectly
60
Q

Severe increase in abnormal erythrocytes in each microscopic field; an equivalent descriptive term is MANY.

0
1+
2+
3+
4+

A

3+

Grading of Erythrocyte Morphology

0 Normal appearance or slight variation in erythrocytes.
1+ Only a small population of erythrocytes displays a particular abnormality; the terms
slightly increased or few would be comparable.
2+ More than occasional numbers of abnormal erythrocytes can be seen in a microscopic
field; an equivalent descriptive term is moderately increased.
3+ Severe increase in abnormal erythrocytes in each microscopic field; an equivalent
descriptive term is many.
4+ The most severe state of erythrocytic abnormality, with the abnormality prevalent
throughout each microscopic field; comparable terms are marked or marked increase.

How well did you know this?
1
Not at all
2
3
4
5
Perfectly
61
Q

Inherited hemolytic anemia due to structural membrane defect:

Thalassemia
Sickle cell anemia
Pyruvate kinase deficiency
Hereditary spherocytosis

A

Hereditary spherocytosis

Examples of Inherited Hemolytic Anemias

STRUCTURAL MEMBRANE DEFECTS
Acanthocytosis
Hereditary spherocytosis
Hereditary elliptocytosis
Hereditary stomatocytosis
Hereditary xerocytosis
Rh null disease

ERYTHROCYTIC ENZYME DEFECTS
G6PD deficiency
Glutathione reductase
Hexokinase
Pyruvate kinase

DEFECTS OF THE HEMOGLOBIN MOLECULE
Hb C disorder
Hb S-C disorder
Hb S-S disorder (sickle cell anemia)
Thalassemia

How well did you know this?
1
Not at all
2
3
4
5
Perfectly
62
Q

Hemoglobinopathies associated with ABNORMAL MOLECULAR STRUCTURE:

Alpha thalassemia
Alpha and beta thalassemia
Sickle cell anemia and beta thalassemia
Sickle cell anemia, sickle cell trait and Hb C disease

A

Sickle cell anemia, sickle cell trait and Hb C disease

Examples of Selected Hemoglobinopathies

ABNORMAL MOLECULAR STRUCTURE
Hb SS (sickle cell anemia)
Hb SA (sickle cell trait)
Hb C disease or trait

RATE OF SYNTHESIS
Beta-Thalassemia
Alpha-Thalassemia

COMBINATION OF TWO MOLECULAR ALTERATIONS OR A MOLECULAR ABNORMALITY
AND SYNTHESIS DEFECT
Hb S–Hb C
Hb S–b-thalassemia

How well did you know this?
1
Not at all
2
3
4
5
Perfectly
63
Q

Elongated and curved nucleus; very clumped chromatin:

Myelocyte
Metamyelocyte
Band
Segmenter neutrophil

A

Band

How well did you know this?
1
Not at all
2
3
4
5
Perfectly
64
Q

Indented nucleus, clumped chromatin:

Promyelocyte
Myelocyte
Metamyelocyte
Band

A

Metamyelocyte

How well did you know this?
1
Not at all
2
3
4
5
Perfectly
65
Q

Mast cells have an appearance similar to that of the blood:

Monocyte
Neutrophil
Eosinophil
Basophil

A

Basophil

How well did you know this?
1
Not at all
2
3
4
5
Perfectly
66
Q

Once the metamyelocyte stage has been reached, cells have undergone ____ cell divisions and the proliferative phase comes to an end.

1 or 2 cell divisions
2 or 3 cell divisions
4 or 5 cell divisions
6 or 7 cell divisions

A

4 or 5 cell divisions

How well did you know this?
1
Not at all
2
3
4
5
Perfectly
67
Q

Nuclear chromatin is coarse and clumped; dark blue (basophilic) cytoplasm around the periphery or in a radial pattern and few cytoplasmic vacuoles:

Type I Downey cells
Type II Downey cells
Type III Downey cells

A

Type II Downey cells

Descriptive Features of the Classic Downey Classification of Lymphocytes Seen in
Infectious Mononucleosis

Type I
Nucleus May be irregularly shaped
Cytoplasm Usually many cytoplasmic vacuoles, dark blue (basophilic)

Type II
Nucleus Chromatin is coarse and clumped
Cytoplasm Increased amount, dark blue (basophilic) around the periphery or in a radial
pattern, a few cytoplasmic vacuoles

Type IIIa
Nucleus Nucleoli usually visible, enlarged in size
Cytoplasm Dark blue (basophilic)

How well did you know this?
1
Not at all
2
3
4
5
Perfectly
68
Q

Myeloid cells demonstrate maturation beyond the blast and promyelocyte stage:

M0 myeloid
M1 myeloid
M2 myeloid
M3 myeloid

A

M2 myeloid

How well did you know this?
1
Not at all
2
3
4
5
Perfectly
69
Q

Abnormal proliferation of both erythroid and granulocytic precursors; may include abnormal megakaryocytic and monocytic proliferations:

M3
M4
M6
M7

A

M6

M6 erythroleukemia
Also known as Di Guglielmo syndrome; abnormal proliferation of both erythroid and
granulocytic precursors; may include abnormal megakaryocytic and monocytic
proliferations

How well did you know this?
1
Not at all
2
3
4
5
Perfectly
70
Q

Small cells predominant; nuclear shape is regular with an occasional cleft; chromatin pattern is homogeneous and nucleoli are rarely visible; cytoplasm is moderately basophilic:

L1
L2
L3

A

L1

L1 homogeneous
One population of cells within the case; small cells predominant; nuclear shape is regular
with an occasional cleft; chromatin pattern is homogeneous and nucleoli are rarely visible;
cytoplasm is moderately basophilic

L2 heterogeneous
Large cells with an irregular nuclear shape; clefts in the nucleus are common; one or more
large nucleoli are visible; cytoplasm varies in color

L3 Burkitt lymphoma type
Cells are large and homogeneous in size; nuclear shape is round or oval; one to three
prominent nucleoli; cytoplasm is deeply basophilic with vacuoles often prominent

How well did you know this?
1
Not at all
2
3
4
5
Perfectly
71
Q

Cells are large and homogeneous in size; nuclear shape is round or oval; one to three prominent nucleoli; cytoplasm is deeply basophilic with vacuoles often prominent:

L1
L2
L3

A

L3

How well did you know this?
1
Not at all
2
3
4
5
Perfectly
72
Q

Leukemic reticuloendotheliosis:

Prolymphocytoc leukemia
Plasma cell leukemia
Hairy cell leukemia
Sezary syndrome

A

Hairy cell leukemia

How well did you know this?
1
Not at all
2
3
4
5
Perfectly
73
Q

Solid tumor counterpart of acute lymphoblastic leukemia:

Lymphoma, undifferentiated
Lymphoma, poorly differentiated leukemia
Lymphoma, well-differentiated leukemia
Chloroma granulocytic leukemia

A

Lymphoma, poorly differentiated leukemia

How well did you know this?
1
Not at all
2
3
4
5
Perfectly
74
Q

Solid tumor counterpart of plasma cell leukemia:

Reticulum cell sarcoma
Chloroma granulocytic leukemia
Myeloma
Lymphoma, undifferentiated

A

Myeloma

How well did you know this?
1
Not at all
2
3
4
5
Perfectly
75
Q

A distinctive feature of the megakaryocyte:

Multinucleated
Multilobular
Multinucleated and multilobular
None of these

A

Multilobular

How well did you know this?
1
Not at all
2
3
4
5
Perfectly
76
Q

A target INR range of ____ is recommended for most indications (e.g., treatment or prophylaxis of deep venous thrombosis [DVT], or prevention of further clotting in patients who have had a myocardial infarction).

INR range of 1.0 to 2.0
INR range of 2.0 to 3.0
INR range of 2.5 to 3.5
INR range of 4.0 to 5.0

A

INR range of 2.0 to 3.0

A target INR range of 2.0 to 3.0 is recommended for most indications (e.g., treatment or
prophylaxis of deep venous thrombosis [DVT], or prevention of further clotting in patients
who have had a myocardial infarction).
An INR of 2.5 to 3.5 is recommended for patients with prosthetic heart valves.
When the INR is used to guide anticoagulant therapy, there are fewer bleeding events.

How well did you know this?
1
Not at all
2
3
4
5
Perfectly
77
Q

The target INR for pulmonary embolism (PE) treatment is ___ for the duration of anticoagulation.

1.0
1.5
2.5
3.0

A

3.0

How well did you know this?
1
Not at all
2
3
4
5
Perfectly
78
Q

The new types of thromboplastins for measuring the PT are mixtures of phospholipids and recombinantly derived _____ tissue factor.

Rabbit
Pig
Horse
Human

A

Human

NEW THROMBOPLASTINS
The new types of thromboplastins for measuring the PT are mixtures of phospholipids and
recombinantly derived human tissue factor.

Because the new thromboplastins are more sensitive (typical ISI, 1.0) than the traditional
North American ones (ISIs, 1.8 to 3.0), the PTs for patients with inherited or acquired
deficiencies of coagulation factors will be much more prolonged with use of the new
reagents, although normal values may change minimally.

How well did you know this?
1
Not at all
2
3
4
5
Perfectly
79
Q

Anticoagulant therapy:

Falsely decreased D-dimer values
Falsely increased D-dimer values
No effect
Cannot be determined

A

Falsely decreased D-dimer values

Conditions That Can Generate Falsely Decreased or Falsely Increased D-Dimer Values
FALSELY DECREASED VALUES
1. Anticoagulant therapy
2. Smaller, older, nonprogressing thrombus

FALSELY INCREASED VALUES
1. Various disease states
2. Post-therapeutic clinical procedures

How well did you know this?
1
Not at all
2
3
4
5
Perfectly
80
Q

Smallest platelets seen:

Wiskott-Aldrich syndrome
May-Hegglin anomaly
Alport syndrome
Bernard-Soulier syndrome

A

Wiskott-Aldrich syndrome

How well did you know this?
1
Not at all
2
3
4
5
Perfectly
81
Q

Glanzmann thrombasthenia and essential athrombia:

Platelet adhesion defect
Primary platelet aggregation defect
Secondary platelet aggregation defect
Isolated platelet factor III deficicency

A

Primary platelet aggregation defect

Hereditary Platelet Function Defects

ADHESION DEFECTS
Bernard-Soulier syndrome
Impaired adhesion to collagen

AGGREGATION DEFECTS: PRIMARY
Glanzmann thrombasthenia
Essential athrombia

AGGREGATION DEFECTS: SECONDARY
Storage pool diseases
Aspirin-like defects
Release reaction defects

ISOLATED PLATELET FACTOR III DEFICIENCY

SEVERE COAGULATION FACTOR DEFICIENCIES
Afibrinogenemia
Factor VIII: C deficiency
Factor IX: C deficiency

How well did you know this?
1
Not at all
2
3
4
5
Perfectly
82
Q

Acquired platelet function defects:

Bernard-Soulier syndrome
Bernard-Soulier and Glanzmann thrombasthenia
Uremia, multiple myeloma
Uremia, multiple myeloma, vitamin B12 or folate deficiency

A

Uremia, multiple myeloma, vitamin B12 or folate deficiency

ACQUIRED PLATELET FUNCTION DEFECTS
1. Myeloproliferative syndromes
Essential thrombocythemia
Chronic myelogenous leukemia
Polycythemia vera
Paroxysmal nocturnal hemoglobinuria
Myelofibrosis
RAEB syndrome
Sideroblastic anemia
2. Paraprotein disorders
Multiple myeloma
Waldenström macroglobulinemia
Essential monoclonal gammopathy
3. Autoimmune diseases
Collagen vascular disease
Antiplatelet antibodies
Immune thrombocytopenias
4. Fibrinogen degradation products
Disseminated intravascular coagulation
Primary fibrinolytic syndromes
Liver disease
5. Anemia
Severe iron deficiency
Severe B12 or folate deficiency
6. Uremia
7. Drug induced
RAEB, refractory anemia with excess blasts

How well did you know this?
1
Not at all
2
3
4
5
Perfectly
83
Q

Inherited platelet dysfunction:

Bernard-Soulier syndrome
Bernard-Soulier syndrome, Glanzmann’s thrombasthenia
Uremia, multiple myeloma
Uremia, multiple myeloma, vitamin B12 or folate deficiency

A

Bernard-Soulier syndrome, Glanzmann’s thrombasthenia

INHERITED PLATELET DYSFUNCTION
1. Surface membrane defects
Bernard-Soulier syndrome
Glanzmann thrombasthenia
Platelet-type von Willebrand disease
2. Defects of granule storage
Alpha-granule deficiency
Gray platelet syndrome
3. Dense granules
Wiskott-Aldrich syndrome
Hermansky-Pudlak syndrome
Chédiak-Higashi syndrome
TAR baby syndrome

How well did you know this?
1
Not at all
2
3
4
5
Perfectly
84
Q

Patients with _____ , the most severe form of von Willebrand disease, are likely to have a major episode of bleeding early in life because significantly decreased amounts of vWF and VIII:C are produced.

Type IA
Type IIB
Type IIC, IID
Type III

A

Type III

How well did you know this?
1
Not at all
2
3
4
5
Perfectly
85
Q

Conditions related to deficiencies of multiple coagulation factors:

Hepatic disease
Hepatic disease and anticoagulant overdose
Anticoagulant overdose and vitamin K deficiency
Hepatic disease, anticoagulant overdose, DIC and vitamin K deficiency

A

Hepatic disease, anticoagulant overdose, DIC and vitamin K deficiency

How well did you know this?
1
Not at all
2
3
4
5
Perfectly
86
Q

Indented or twisted nucleus, lacy chromatin and gray-blue cytoplasm:

Segmented neutrophil
Band neutrophil
Monocyte
Lymphocyte

A

Monocyte

How well did you know this?
1
Not at all
2
3
4
5
Perfectly
87
Q

Forward high-angle light scatter:

0 degree angle
2 to 3 degree angle
5 to 15 degree angle
90 degree angle

A

5 to 15 degree angle

Angles of Light Scatter
Various angles of light scatter can aid in cellular analysis.
1. Forward light scatter 0°. This is diffracted light, which relates to the volume of the cell.
2. Forward low-angle light scatter 2° to 3°. This characteristic can relate to size or volume.
3. Forward high angle 5° to 15°. This type of measurement allows for description of the
refractive index of cellular components.
4. Orthogonal light scatter 90°. The result of this application of light scatter is the
production of data based on reflection and refraction of internal components, which
correlates with internal complexity

How well did you know this?
1
Not at all
2
3
4
5
Perfectly
88
Q

RBC histogram to the LEFT:

RBCs are larger than normal
RBCs are smaller than normal
Seen in megaloblastic anemia
Treated anemia

A

RBCs are smaller than normal

If the cells are smaller than normal, the curve will be more to the left, as in untreated iron
deficiency anemia.
If the cells are larger than normal, the histogram curve will be more to the right, as in the
megaloblastic anemias.
After appropriate treatment of the underlying cause of an anemia, the curve should move
toward the normal range.

How well did you know this?
1
Not at all
2
3
4
5
Perfectly
89
Q

Erythrocytes with an increased RDW;

Homogenous in character, very little anisocytosis
Homogenous, high degree of anisocytosis
Heterogenous, very little anisocytosis
Heterogenous, high degree of anisocytosis

A

Heterogenous, high degree of anisocytosis

Erythrocytes with a normal RDW are homogeneous in character and exhibit very little
anisocytosis on a peripheral blood smear.

Erythrocytes with an increased RDW are referred to as heterogeneous and exhibit a high
degree of anisocytosis on a peripheral blood smear.

How well did you know this?
1
Not at all
2
3
4
5
Perfectly
90
Q

MPV values should be based on specimens that are between ____ hours old.

1 and 4 hours old
4 and 8 hours old
6 and 12 hours old
12 and 16 hours old

A

1 and 4 hours old

MPV is a measure of the average volume of platelets in a sample.

In EDTA–anti-coagulated blood, platelets undergo a change in shape. This alteration
(swelling) causes the MPV to increase approximately 20% during the first hour. After this
time, the size is stable for at least 12 hours; however, MPV values should be based on
specimens that are between 1 and 4 hours old.

No single normal range exists. Patients with a lower platelet count normally have a higher
MPV, and patients with a higher platelet count have a lower MPV.

Analysis of a nomogram demonstrates that an MPV between 9.0 and 9.8 fL is in the
normal range, if the platelet count is normal. MPVs from 7.8 to 8.9 fL or from 9.9 to 12.0 fL
may be in the normal range, depending on the platelet count.

DECREASED MPV:
1. Aplastic anemia
2. Megaloblastic anemia
3. Wiskott-Aldrich syndrome
4. After chemotherapy

INCREASED MPV:
1. Idiopathic thrombocytopenic purpura
2. After splenectomy
3. Sickle cell anemia

How well did you know this?
1
Not at all
2
3
4
5
Perfectly
91
Q

Measure of the uniformity of platelet size in a blood specimen:

Platelet adhesion
Platelet aggregation
Mean platelet volume (MPV)
Platelet distribution width (PDW)

A

Platelet distribution width (PDW)

How well did you know this?
1
Not at all
2
3
4
5
Perfectly
92
Q

Placement of fire extinguishers every ___ feet.

75 feet
100 feet
125 feet
150 feet

A

75 feet

Placement of fire extinguishers every 75 feet. A distinct system for marking the locations
of fire extinguishers enables quick access when they are needed. Fire extinguishers
should be checked monthly and maintained annually.
Placement of manual fire alarm boxes near the exit doors. Travel distance should not
exceed 200 feet.

How well did you know this?
1
Not at all
2
3
4
5
Perfectly
93
Q

Surfaces in the specimen collection and processing area should be cleaned with:

70% isopropyl alcohol.
1:10 bleach solution.
Soap and water.
Any of the above

A

1:10 bleach solution.

How well did you know this?
1
Not at all
2
3
4
5
Perfectly
94
Q

Which of the following is a proper way to clean up a small blood spill that has dried on a countertop?

Moisten it with a disinfectant and carefully absorb it with a paper towel.
Rub it with an alcohol pad, then wipe the area with a clean alcohol pad.
Scrape it into a biohazard bag and wash the surface with soap and water.
Use a disinfectant wipe and scrub it in ever-increasing concentric circles

A

Moisten it with a disinfectant and carefully absorb it with a paper towel.

How well did you know this?
1
Not at all
2
3
4
5
Perfectly
95
Q

The following test orders for different patient shave been received at the same time. Which test would you collect first?

Fasting glucose
STAT glucose in the ER
STAT hemoglobin in ICU
ASAP CBC in ICU

A

STAT glucose in the ER

How well did you know this?
1
Not at all
2
3
4
5
Perfectly
96
Q

A member of the clergy is with the patient when you arrive to collect a routine specimen. What should you do?

Ask the patient’s nurse what you should do.
Come back after the clergy person has gone.
Fill out a form saying you were unable to collect the specimen.
Say “Excuse me, I need to collect a specimen from this patient.”

A

Come back after the clergy person has gone.

If a physician or a member of the clergy is with the patient, don’t interrupt. The patient’s
time with these individuals is private and limited. If the draw is not stat, timed or other
urgent priority, go draw another patient and check back after that. If that is the only
patient, wait outside the room for a few minutes or go back to the lab and draw the
specimen on the next sweep. (In any case, always make certain your actions follow facility
policy.) If the request is stat, timed, or other urgent priority , excuse yourself, explain why
you are there, and ask permission to proceed.

How well did you know this?
1
Not at all
2
3
4
5
Perfectly
97
Q

The most common complication encountered in obtaining a blood specimen; it is caused by leakage of a small amount of blood in the tissue around the puncture site:

Petechiae
Hematoma
Ecchymosis
Hemoconcentration

A

Ecchymosis

Ecchymosis (Bruise): Bruising is the most common complication encountered in obtaining
a blood specimen. It is caused by leakage of a small amount of blood in the tissue around
the puncture site.
Hematoma: A hematoma results when leakage of a large amount of blood around the
puncture site causes the area to rapidly swell.

How well did you know this?
1
Not at all
2
3
4
5
Perfectly
98
Q

A patient complains of extreme pain when you insert the needle during a venipuncture attempt. The pain does not subside, but the patient does not feel any numbness or burning sensation. You know the needle is in the vein because the blood is flowing into the tube. You have only two tubes to fill, and the first one is almost full. What should you do?

Ask the patient if he or she wants you to continue the draw
Discontinue the draw and attempt collection at another site
Distract the patient with small talk and continue the draw
Tell the patient to hang in there as you have only one tube left

A

Discontinue the draw and attempt collection at another site

If marked or extreme pain occurs, or the patient asks you to remove the needle for any
reason, the venipuncture should be terminated immediately, even if there are no other
signs of nerve injury.

How well did you know this?
1
Not at all
2
3
4
5
Perfectly
99
Q

Which type of patient is most likely to have an arteriovenous fistula or graft?

Arthritic
Dialysis
Hospice
Wheelchair-bound

A

Dialysis

An arteriovenous (AV) shunt, fistula, or graft is the permanent surgical connection of an
artery and vein by direct fusion (fistula), resulting in a bulging vein, or with a piece of vein
or tubing (graft) that creates a loop under the skin. It is typically created to be used for
dialysis, commonly joins the radial artery and cephalic vein above the wrist on the
underside of the arm, and has a distinctive buzzing sensation called a “thrill” when
palpated. A temporary shunt with tubing on the surface of the skin can also be created.

How well did you know this?
1
Not at all
2
3
4
5
Perfectly
100
Q

Type of immersion oil with high viscosity and is used in brightfield and standard clinical microscopy. In hematology, this oil is routinely used.

Type A
Type B
Type C

A

Type B

Three types of immersion oil, differing in viscosity, are employed in the clinical laboratory:
1. Type A has very low viscosity and is used in fluorescence and darkfield studies.
2. Type B has high viscosity and is used in brightfield and standard clinical microscopy. In
hematology, this oil is routinely used.
3. Type C has very high viscosity and is used with inclined microscopes with long-focus
objective lenses and wide condenser gaps.

How well did you know this?
1
Not at all
2
3
4
5
Perfectly
101
Q

The recommended cleaner for removing oil from objectives is:

Benzene
Xylene
Water
70% alcohol or lens cleaner

A

70% alcohol or lens cleaner

Use solvent sparingly. The use of xylene is discouraged, because it contains a
carcinogenic component (benzene). Xylene is also a poor cleaning agent, leaving an oily
film on the lens. Lens cleaner or 70% isopropyl alcohol employed sparingly on a cotton
applicator stick can be used to clean the objective lenses.

How well did you know this?
1
Not at all
2
3
4
5
Perfectly
102
Q

Often the objects appear to have “haloes” surrounding them.

Brightfield microscope
Darkfield microscope
Phase-contrast microscope
Polarized light microscope

A

Phase-contrast microscope

How well did you know this?
1
Not at all
2
3
4
5
Perfectly
103
Q

True for PRECISION:

Measure of agreement between an assay value and the theoretical “true value” of its analyte
Magnitude of error separating the assay result from the true value
Easy to define but difficult to establish and maintain
Relatively easy to measure and maintain

A

Relatively easy to measure and maintain

Accuracy is easy to define but difficult to establish and maintain; precision is relatively
easy to measure and maintain.
Precision is the expression of reproducibility or dispersion about the mean, often
expressed as SD or CV%.

How well did you know this?
1
Not at all
2
3
4
5
Perfectly
104
Q

Slope measures:

Random error
Constant systematic error
Proportional systematic error
Constant and proportional systematic error

A

Proportional systematic error

Perfect correlation generates a slope of 1 and a y intercept of 0.

Slope measures proportional systematic error; the higher the analyte value, the greater the
deviation from the line of identity. Proportional errors are caused by malfunctioning
instrument components or a failure of some part of the testing process. The magnitude of
the error increases with the concentration or activity of the analyte. An assay with
proportional error may be invalid.

Intercept measures constant systematic error (or bias, in laboratory vernacular), a
constant difference between the new and reference assay regardless of assay result
magnitude. A laboratory director may choose to adopt a new assay with systematic error
but must modify the published reference interval.

How well did you know this?
1
Not at all
2
3
4
5
Perfectly
105
Q

The positive predictive value predicts the probability that an individual with a positive assay result ___ the disease or condition.

Has
Could have
May have
Will have

A

Has

The positive predictive value predicts the probability that an individual with a positive
assay result has the disease or condition.
The negative predictive value predicts the probability that an individual with a negative
assay result does not have the disease or condition.

How well did you know this?
1
Not at all
2
3
4
5
Perfectly
106
Q

It describes the total number of events or conditions in a broadly defined population, for instance, the total number of patients with chronic heart disease in the Philippines.

Incidence
Prevalence
False negative
False positive

A

Prevalence

Epidemiologists describe population events using the terms prevalence and incidence.
1. Prevalence describes the total number of events or conditions in a broadly defined
population, for instance, the total number of patients with chronic heart disease in the United States.
2. Incidence describes the number of events occurring within a randomly selected number
of subjects representing a population, over a defined time, for instance, the number of newcases of heart disease per 100,000 U.S. residents per year.

Scientists use incidence, not prevalence, to select laboratory assays for specific
applications such as screening or confirmation.

How well did you know this?
1
Not at all
2
3
4
5
Perfectly
107
Q

Type of chromatin represented by the more darkly stained, condensed clumping pattern and is the transcriptionally inactive area of the nucleus

Euchromatin
Heterochromatin

A

Heterochromatin

Morphologically, chromatin is divided into two types:
(1) the heterochromatin, which is represented by the more darkly stained, condensed
clumping pattern and is the transcriptionally
inactive area of the nucleus
(2) the euchromatin, which has diffuse, uncondensed, open chromatin and is the genetically active portion of the nucleus where DNA transcription into mRNA
occurs. The euchromatin is loosely coiled and turns a pale blue when stained with Wright
stain.
More mature cells have more heterochromatin because they are less transcriptionally
active.

How well did you know this?
1
Not at all
2
3
4
5
Perfectly
108
Q

In ____, the tetraploid DNA is checked for proper replication and damage takes approximately 4 hours.

G1
S
G2
M
G0

A

G2

The cell cycle is a biochemical and morphologic four-stage process through which a cell
passes when it is stimulated to divide.

These stages are G1 (gap 1), S (DNA synthesis), G2 (gap 2), and M (mitosis).
G1 is a period of cell growth and synthesis of components necessary for replication. G1 lasts about 10 hours.

In the S stage, DNA replication takes place, a process requiring about 8 hours. An exact
copy of each chromosome is produced and they pair together as sister chromatids. The
centrosome is also duplicated during the S stage.

In G2, the tetraploid DNA is checked for proper replication and damage. G2 takes
approximately 4 hours.

The time spent in each stage can be variable, but mitosis takes approximately 1 hour.

During G0 (quiescence) the cell is not actively in the cell cycle.

How well did you know this?
1
Not at all
2
3
4
5
Perfectly
109
Q

All of the following statements refers to APOPTOSIS, except:

Enlarged cell size due to swelling
Reduced cell size due to shrinkage
Condensation and fragmentation of the nucleus between nucleosomes
Mostly physiologic to remove unwanted cells

A

Enlarged cell size due to swelling

APOPTOSIS
Reduced due to shrinkage
Condensation and fragmentation between nucleosomes
Mostly physiologic to remove unwanted cells; pathologic in response to cell injury
—-
NECROSIS
Enlarged due to swelling
Random breaks and lysis (karyolysis)
Pathologic; results from cell injury

How well did you know this?
1
Not at all
2
3
4
5
Perfectly
110
Q

The process of replacing the active marrow by adipocytes (yellow marrow) during development is

Hematopoiesis
Progression
Regression
Retrogression

A

Retrogression

The process of replacing the active marrow by adipocytes (yellow marrow) during
development is called retrogression and eventually results in restriction of the active
marrow in the adult to the sternum, vertebrae, scapulae, pelvis, ribs, skull, and proximal
portion of the long bones.

How well did you know this?
1
Not at all
2
3
4
5
Perfectly
111
Q

The major site of blood cell production during the second trimester of fetal development.

Yolk sac
Liver
Spleen
Bone marrow

A

Liver

The liver serves as the major site of blood cell production during the second trimester of
fetal development.
In adults, the hepatocytes of the liver have many functions, including protein synthesis and
degradation, coagulation factor synthesis, carbohydrate and lipid metabolism, drug and
toxin clearance, iron recycling and storage, and hemoglobin degradation in which bilirubin
is conjugated and transported to the small intestine for eventual excretion

How well did you know this?
1
Not at all
2
3
4
5
Perfectly
112
Q

The largest lymphoid organ in the body:

Bone marrow
Thymus
Liver
Spleen

A

Spleen

How well did you know this?
1
Not at all
2
3
4
5
Perfectly
113
Q

Cytokines that function for STEM CELL MOBILIZATION:

IL-1 and IL-2
IL-2 and IFN-alpha
IL-12 and IL-15
IL-3, G-CSF and GM-CSF

A

IL-3, G-CSF and GM-CSF

How well did you know this?
1
Not at all
2
3
4
5
Perfectly
114
Q

In adults, hematopoietic tissue is located in the:

Bone marrow
Bone marrow and lymph nodes
Bone marrow, lymph nodes, liver and spleen
Bone marrow, lymph nodes, spleen, liver and thymus

A

Bone marrow, lymph nodes, spleen, liver and thymus

How well did you know this?
1
Not at all
2
3
4
5
Perfectly
115
Q

In aplastic anemia, the bone marrow is:

Empty
Empty, hypoplastic
Empty, hyperplastic
Either hypoplastic or hyperplastic

A

Empty, hypoplastic

How well did you know this?
1
Not at all
2
3
4
5
Perfectly
116
Q

Second step in phagocytosis:

Recognition and attachement
Ingestion
Killing and digestion
Formation of neutrophil extracellular trap

A

Ingestion

PHAGOCYTOSIS (RODAK)
1. Recognition and attachment
2. Ingestion
3. Killing and digestion
4. Formation of neutrophil extracellular trap

How well did you know this?
1
Not at all
2
3
4
5
Perfectly
117
Q

The promonocyte nucleus is deeply indented and should not be confused with a:

Lymphocyte
Erythrocyte
Segmenter neutrophil
Band neutrophil

A

Band neutrophil

How well did you know this?
1
Not at all
2
3
4
5
Perfectly
118
Q

Which of the following cells does not exhibit myeloperoxidase (MPO) activity?

Neutrophils
Eosinophils
Monocytes
Lymphocytes

A

Lymphocytes

Myeloperoxidase (MPO) is an enzyme found in the primary granules of granulocytic cells
(neutrophils, eosinophils, and, to a certain extent, monocytes). Lymphocytes do not exhibit
MPO activity. This stain is useful for differentiating the blasts of acute myeloid leukemia
(AML) from those of acute lymphoblastic leukemia (ALL).

How well did you know this?
1
Not at all
2
3
4
5
Perfectly
119
Q

NEWER TECHNIQUES USED IN THE DIAGNOSIS of acute leukemias:

Morphology and cytochemistry
Cytochemistry and cytogenetics
Flow cytometry and cytogenetic analysis
Flow cytometry, cytogenetic analysis and molecular testing

A

Flow cytometry, cytogenetic analysis and molecular testing

How well did you know this?
1
Not at all
2
3
4
5
Perfectly
120
Q

Basophilic and granular cytoplasm

MK-I
MK-II
MK-III

A

MK-II

How well did you know this?
1
Not at all
2
3
4
5
Perfectly
121
Q

Differentiation stage(s) characterized by presence of demarcation system:

MK-I
MK-I and MK-II
MK-II and MK-III
MK-I, MK-II and MK-III

A

MK-I, MK-II and MK-III

How well did you know this?
1
Not at all
2
3
4
5
Perfectly
122
Q

Cytokine(s) that function to stimulate megakaryocytopoiesis:

Thrombopoietin (TPO)
TPO and IL-3
TPO, IL-3 and IL-6
TPO, IL-3, IL-6 and IL-11

A

TPO, IL-3, IL-6 and IL-11

How well did you know this?
1
Not at all
2
3
4
5
Perfectly
123
Q

Demonstrates the largest platelets seen and is also referred to as giant platelet syndrome:

Epstein syndrome
Mediterranean macrothrombocytopenia
May-Hegglin anomaly
Bernard-Soulier syndrome

A

Bernard-Soulier syndrome

How well did you know this?
1
Not at all
2
3
4
5
Perfectly
124
Q

Ratio of blood to anticoagulant for coagulation testing:

1:4
1:9
4:1
9:1

A

9:1

How well did you know this?
1
Not at all
2
3
4
5
Perfectly
125
Q

Hemostasis specimen STORAGE temperature:

1 to 6 C
18 to 24 C
36.5 to 37.5 C
30 to 37 C

A

18 to 24 C

HEMOSTASIS SPECIMEN STORAGE TEMPERATURE
Sodium citrate-anticoagulated whole blood specimens are placed in a rack and allowed to
stand in a vertical position with the stopper intact and uppermost. The pH remains
constant as long as the specimen is sealed. Specimens are maintained at 18° C to 24° C
(ambient temperature), never at refrigerator temperatures.

Storage at 1° C to 6° C activates factor VII, destroys platelet activity through uncontrolled
activation, and causes the cryoprecipitation of large VWF multimers. Also, specimens
should never be stored at temperatures greater than 24° C because heat causes
deterioration of coagulation factors V and VIII.

How well did you know this?
1
Not at all
2
3
4
5
Perfectly
126
Q

Most coagulation studies are carried out at which temperature?

-20C
-70C
24C
37C

A

37C

How well did you know this?
1
Not at all
2
3
4
5
Perfectly
127
Q

Possible solution when specimen is icteric or lipemic for a clot-based test:

PT falsely shortened; recollect specimen.
PT falsely prolonged; recollect specimen.
Measure PT using a mechanical coagulometer
Adjust anticoagulant volume

A

Measure PT using a mechanical coagulometer

How well did you know this?
1
Not at all
2
3
4
5
Perfectly
128
Q

Possible effect and solution when blood collection volume is less than the specified minimum for a clot-based test:

PT falsely shortened; recollect specimen
PT falsely prolonged; recollect specimen.
Use reagent known to be insensitive to heparin
Use chromogenic factor X assay instead of PT

A

PT falsely prolonged; recollect specimen.

How well did you know this?
1
Not at all
2
3
4
5
Perfectly
129
Q

Test that assess deficiencies of all factors except VII and XIII:

Prothrombin time (PT)
Partial thromboplastin time (PTT)
Thrombin time (TT)
Reptilase time

A

Partial thromboplastin time (PTT)

How well did you know this?
1
Not at all
2
3
4
5
Perfectly
130
Q

Part of the INITIAL VON WILLEBRAND DISEASE WORKUP:

BT, PT and APTT
CBC, PT and APTT
CBC, BT, PT and APTT
CBC, BT, PT, APTT and automated functional platelet assays
Decreased vWF activity and personal/family history of mucocutaneous bleeding

A

CBC, PT and APTT

How well did you know this?
1
Not at all
2
3
4
5
Perfectly
131
Q

DEFINITIVE DIAGNOSIS OF VON WILLEBRAND DISEASE:

BT, PT and APTT
CBC, PT and APTT
CBC, BT, PT and APTT
CBC, BT, PT, APTT and automated functional platelet assays
Laboratory demonstration of decreased vWF activity
Decreased vWF activity and personal/family history of mucocutaneous bleeding

A

Decreased vWF activity and personal/family history of mucocutaneous bleeding

How well did you know this?
1
Not at all
2
3
4
5
Perfectly
132
Q

In Coulter instruments, which parameters are directly measured:

RBC count and hemoglobin
RBC and WBC counts
RBC count and hematocrit
RBC and WBC counts, hemoglobin

A

RBC and WBC counts, hemoglobin

How well did you know this?
1
Not at all
2
3
4
5
Perfectly
133
Q

Lipemia, icterus:

Increased Hb
Increased Hb, decreased MCH
Increased Hb and MCH
Decreased Hb and MCH

A

Increased Hb and MCH

How well did you know this?
1
Not at all
2
3
4
5
Perfectly
134
Q

Platelet clumps:

Decreased platelets
Decreased platelets and WBCs
Decreased platelets, increased WBCs
Increased platelets and WBCs

A

Decreased platelets, increased WBCs

How well did you know this?
1
Not at all
2
3
4
5
Perfectly
135
Q

Parameters affected when the WBC count > 100,000/uL:

Increased RBCs, decreased hemoglobin
Decreased RBCs, increased hemoglobin
Decreased RBCs and hemoglobin
Increased RBcs and hemoglobin, incorrect hematocrit

A

Increased RBcs and hemoglobin, incorrect hematocrit

How well did you know this?
1
Not at all
2
3
4
5
Perfectly
136
Q

Compare the volume of plasma to serum obtained from a given volume of whole blood:

Plasma greater volume than serum
Plasma lesser volume than serum
Same volume
Variable

A

Plasma greater volume than serum

How well did you know this?
1
Not at all
2
3
4
5
Perfectly
137
Q

EDTA is used in concentrations of ___ mg/1 mL of whole blood.

0.5 mg/1 mL of whole blood
1.5 mg/1 mL of whole blood
2.0 mg/1 mL of whole blood
2.5 mg/1 mL of whole blood

A

1.5 mg/1 mL of whole blood

How well did you know this?
1
Not at all
2
3
4
5
Perfectly
138
Q

A 7.0 mL EDTA tube is received in the laboratory containing only 2.0 mL of blood. If the laboratory is using manual techniques, which of the following tests will most likely be erroneous?

A. RBC count
B. Hemoglobin
C. Hematocrit
D. WBC count

A

C. Hematocrit

How well did you know this?
1
Not at all
2
3
4
5
Perfectly
139
Q

EDTA-induced pseudothrombocytopenia can be identified on blood smear by:

A. Finding platelets pushed to the feathered end
B. Finding platelets adhering to WBCs
C. Finding no platelets at all on the smear
D. Bluish discoloration to the macroscopic appearance of the slide

A

B. Finding platelets adhering to WBCs

Platelet satellitosis (platelet encircling the peripheral borders of neutrophils) is seen in a
rare patient whose blood is anticoagulated with EDTA. This phenomenon is thought to be
due to a serum factor which reacts in the presence of EDTA.

How well did you know this?
1
Not at all
2
3
4
5
Perfectly
140
Q

The automated platelet count on an EDTA specimen is 58 x 10 9th/L. The platelet estimate on the blood smear appears normal, but it was noted that the platelets were surrounding the neutrophils. The next step should be to:

A. Report the automated platelet count since it is more accurate than a platelet estimate
B. Warm the EDTA tube and repeat the automated platelet count
C. Rerun the original specimen since the platelet count and blood smear estimate do not match
D. Recollect a specimen for a platelet count using a different anticoagulant

A

D. Recollect a specimen for a platelet count using a different anticoagulant

How well did you know this?
1
Not at all
2
3
4
5
Perfectly
141
Q

Sodium citrate in the concentration of ___ solution has been adopted as the appropriate concentration for coagulation studies.

A. 1.5%
B. 2.8%
C. 3.2%
D. 3.8%

A

C. 3.2%

Sodium citrate in the concentration of a 3.2% solution has been adopted as the
appropriate concentration by the ICSH and the International Society for Thrombosis and
Hemostasis for coagulation studies.

How well did you know this?
1
Not at all
2
3
4
5
Perfectly
142
Q

Which ratio of anticoagulant to blood is correct for coagulation procedures?

A. 1:4
B. 1:5
C. 1:9
D. 1:10

A

C. 1:9

How well did you know this?
1
Not at all
2
3
4
5
Perfectly
143
Q

Which results would be expected for the PT and APTT in a patient with polycythemia?

A. Both prolonged
B. Both shortened
C. Normal PT, prolonged APTT
D. Both normal

A

A. Both prolonged

How well did you know this?
1
Not at all
2
3
4
5
Perfectly
144
Q

What is the proper angle of needle insertion for phlebotomy?

A. 5 degrees
B. 15 degrees
C. 35 degrees
D. 45 degrees

A

B. 15 degrees

How well did you know this?
1
Not at all
2
3
4
5
Perfectly
145
Q

Select the needle most commonly used in standard venipuncture in an adult:

A. One inch, 18 gauge
B. One inch, 21 gauge
C. One-half inch, 21 gauge
D. One-half inch, 25 gauge

A

B. One inch, 21 gauge

How well did you know this?
1
Not at all
2
3
4
5
Perfectly
146
Q

The bevel of the needle should be held _____ in the performance of a venipuncture.

A. Sideways
B. Upward
C. Downward
D. In any direction

A

B. Upward

How well did you know this?
1
Not at all
2
3
4
5
Perfectly
147
Q

Most common complication encountered in obtaining a blood specimen:

A. Ecchymosis (bruise)
B. Hematoma
C. Hemoconcentration
D. Anemia

A

A. Ecchymosis (bruise)

How well did you know this?
1
Not at all
2
3
4
5
Perfectly
148
Q

It is caused by leakage of a SMALL AMOUNT OF BLOOD in the tissue around the puncture site:

A. Ecchymosis (bruise)
B. Hematoma
C. Hemoconcentration
D. Anemia

A

A. Ecchymosis (bruise)

How well did you know this?
1
Not at all
2
3
4
5
Perfectly
149
Q

Leakage of a LARGE AMOUNT OF BLOOD around the puncture site causes the area to rapidly swell:

A. Ecchymosis (bruise)
B. Hematoma
C. Hemoconcentration
D. Anemia

A

B. Hematoma

How well did you know this?
1
Not at all
2
3
4
5
Perfectly
150
Q

VASCULAR COMPLICATIONS of phlebotomy:

A. Bleeding from the site of the venipuncture and hematoma
B. Pseudoaneurysm, thrombosis
C. Reflex arteriospasm, arteriovenous fistula formation
D. All of these

A

D. All of these

Bleeding from the site of the venipuncture and hematoma formation are the most
common vascular complications.
Uncommon vascular complications that are not usually related to the technique include
pseudoaneurysm, thrombosis, reflex arteriospasm, and arteriovenous fistula formation.

How well did you know this?
1
Not at all
2
3
4
5
Perfectly
151
Q

CARDIOVASCULAR COMPLICATIONS of phlebotomy:

A. Orthostatic hypotension
B. Syncope
C. Shock and cardiac arrest
D. All of these

A

D. All of these

How well did you know this?
1
Not at all
2
3
4
5
Perfectly
152
Q

A blood sample is needed from a patient with IV fluids running in both arms. Which of the following is an acceptable procedure?

A. Any obtainable vein is satisfactory.
B. Obtain sample from above the IV site.
C. Obtain sample from below the IV site with special restrictions.
D. Disconnect the IV line.
E. Do not draw a blood specimen

A

C. Obtain sample from below the IV site with special restrictions.

How well did you know this?
1
Not at all
2
3
4
5
Perfectly
153
Q

When encountering a patient with a FISTULA, the phlebotomist should:

Apply the tourniquet below the fistula
Use the other arm
Collect the blood from the fistula
Attach a syringe to the T-tube connector

A

Use the other arm

FISTULA: Permanent surgical connection between an artery and a vein (used for dialysis)
CANNULA: Tube that can be inserted into a cavity

How well did you know this?
1
Not at all
2
3
4
5
Perfectly
154
Q

If a patient adamantly refuses to have blood drawn, you should:

Convince the patient to be cooperative
Notify the patient’s nurse or physician
Restrain the patient and draw the blood
Write a note to the patient’s physician

A

Notify the patient’s nurse or physician

How well did you know this?
1
Not at all
2
3
4
5
Perfectly
155
Q

Blood collection tubes are labeled:

As soon as the test order is received
Before the specimen is even collected
Immediately after specimen collection
After returning to the laboratory

A

Immediately after specimen collection

How well did you know this?
1
Not at all
2
3
4
5
Perfectly
156
Q

Which of the following is a proper way to clean up a small blood spill that has dried on a countertop?

Moisten it with a disinfectant and carefully absorb it with a paper towel
Rub it with an alcohol pad, then wipe the area with a clean alcohol pad
Scrape it into a biohazard bag and wash the surface with soap and water
Use a disinfectant wipe and scrub it in ever increasing concentric circles

A

Moisten it with a disinfectant and carefully absorb it with a paper towel

How well did you know this?
1
Not at all
2
3
4
5
Perfectly
157
Q

The appropriate dilution of bleach to be used in laboratory disinfection is:

1:2
1:5
1:10
1:100

A

1:10

How well did you know this?
1
Not at all
2
3
4
5
Perfectly
158
Q

Which order of events should be followed at the conclusion of a laboratory worker’s shift in order to prevent the spread of bloodborne pathogens?

Remove gloves, disinfect area, wash hands, remove lab coat
Disinfect area, remove gloves, remove lab coat, wash hands
Disinfect area, remove gloves, wash hands, remove lab coat
Remove gloves, wash hands, remove lab coat, disinfect area

A

Disinfect area, remove gloves, remove lab coat, wash hands

According to the OSHA Bloodborne Pathogens Rule of 1992, gloves and lab coats are to
be removed after disinfection of the work area.

How well did you know this?
1
Not at all
2
3
4
5
Perfectly
159
Q

Isolation techniques:

Prevent spread of infection from patient to hospital personnel
Prevent spread of infection from patient to other patients
Protect infection prone patient from pathogens
All of these

A

All of these

How well did you know this?
1
Not at all
2
3
4
5
Perfectly
160
Q

In ENTERIC ISOLATION, the technologist is required to wear

Gown and gloves
Gown, mask and gloves
Gown, mask, gloves and shoe coverings
Mask

A

Gown and gloves

STRICT ISOLATION: Gown, mask and gloves
ENTERIC ISOLATION: Gown and gloves
RESPIRATORY ISOLATION: Mask, gloves
WOUND AND SKIN ISOLATION: Gown and gloves
PROTECTIVE ISOLATION: Gown, mask, gloves, shoe coverings

How well did you know this?
1
Not at all
2
3
4
5
Perfectly
161
Q

Reverse isolation may be used for:

A patient with the measles
An adult patient with the flu
A patient with tuberculosis
A patient with severe burns

A

A patient with severe burns

Patients requiring PROTECTIVE ISOLATION are those with compromised immune systems,
such as neutropenic patients (those with abnormally low white blood cell counts); severely
burned patients; and patients with compromised immune systems, such as patients with
AIDS.

How well did you know this?
1
Not at all
2
3
4
5
Perfectly
162
Q

The first hemostatic response to injury of a blood vessel is:

Platelet adhesion
Platelet aggregation
Vasoconstriction
Extrinsic coagulation

A

Vasoconstriction

How well did you know this?
1
Not at all
2
3
4
5
Perfectly
163
Q

The enzyme inhibited by aspirin is:

Thromboxane synthetase
Cyclooxygenase
Lactate dehydrogenase
Phospholipase

A

Cyclooxygenase

How well did you know this?
1
Not at all
2
3
4
5
Perfectly
164
Q

The life span of a platelet is about:

2 to 3 hours
1 to 3 days
8 to 11 days
60 to 80 days

A

8 to 11 days

How well did you know this?
1
Not at all
2
3
4
5
Perfectly
165
Q

Approximately ___ of the total number of platelets circulate in the systemic circulation?

One-fourth
One-third
One-half
Two-thirds

A

Two-thirds

How well did you know this?
1
Not at all
2
3
4
5
Perfectly
166
Q

The normal range of platelets in the systemic circulation is:

50 - 150 x 10 9th/L
100 - 200 x 10 9th/L
150 - 400 x 10 9th/L
Greater than 500 x 10 9th/L

A

150 - 400 x 10 9th/L

How well did you know this?
1
Not at all
2
3
4
5
Perfectly
167
Q

Effect of platelet clumps to automated cell counting:

Decreased platelets and WBCs
Increased platelets and WBCs
Decreased platelets, increased WBCs
Increased platelets, decreased WBCs

A

Decreased platelets, increased WBCs

RATIONALE: Large clumps counted as WBCs and not platelets
CORRECTIVE ACTION: Redraw specimen in sodium citrate, multiply result by 1.1

How well did you know this?
1
Not at all
2
3
4
5
Perfectly
168
Q

In disseminated intravascular coagulation (DIC) and immune thrombocytopenic purpura (ITP):

There is decreased production of platelets
There is increased destruction of platelets
There is a defect of platelet membrane
There is defect of platelet release reaction

A

There is increased destruction of platelets

How well did you know this?
1
Not at all
2
3
4
5
Perfectly
169
Q

Immune thrombocytopenic purpura (ITP):

Formerly known as disseminated intravascular coagulation (DIC)
Absence of megakaryocytes in the bone marrow
Widespread formation of platelet thrombi
Due to platelet antibodies

A

Due to platelet antibodies

How well did you know this?
1
Not at all
2
3
4
5
Perfectly
170
Q

In thrombocythemia, the platelets are:

Increased
Decreased
Normal
Normal in number, abnormal morphology

A

Increased

How well did you know this?
1
Not at all
2
3
4
5
Perfectly
171
Q

Which of the following is characteristic of Bernard-Soulier syndrome?

Giant platelets
Normal bleeding time
Abnormal aggregation with ADP
Increased platelet count

A

Giant platelets

How well did you know this?
1
Not at all
2
3
4
5
Perfectly
172
Q

Platelet aggregation studies revealed normal aggregation curves with collagen, epinephrine, and ADP, but an abnormal aggregation curve with ristocetin. Based on these findings, what is the differential diagnosis?

-Von Willebrand disease and Bernard-Soulier syndrome
-Glanzmann’s thrombasthenia and von Willebrand disease
-Storage pool disease and Glanzmann’s thrombasthenia
-Bernard-Soulier syndrome and storage pool disease

A

Von Willebrand disease and Bernard-Soulier syndrome

How well did you know this?
1
Not at all
2
3
4
5
Perfectly
173
Q

Which set of platelet responses would be most likely be associated with Glanzmann’s thrombasthenia?

-Normal platelet aggregation response to ADP and ristocetin; decreased response to collagen
-Normal platelet aggregation response to collagen; decreased response to ADP and collagen
-Normal platelet aggregation response to ristocetin; decreased response to collagen, ADP and epinephrine
-Normal platelet aggregation response to ADP; decreased response to collagen and ristocetin

A

Normal platelet aggregation response to ristocetin; decreased response to collagen, ADP and epinephrine

How well did you know this?
1
Not at all
2
3
4
5
Perfectly
174
Q

Primary PLATELET AGGREGATION disorders:

Bernard-Soulier syndrome
Glanzmann’s thrombasthenia
Essential athrombia
Glanzmann’s thrombasthenia and essential athrombia

A

Glanzmann’s thrombasthenia and essential athrombia

Glanzmann thrombasthenia and essential athrombia are similar, rare, primary aggregation
disorders.

How well did you know this?
1
Not at all
2
3
4
5
Perfectly
175
Q

To evaluate normal platelet numbers in an appropriate area of a blood smear, approximately how many platelets, should be observed per oil immersion field?*

1 to 4
8 to 20
4 to 10
20 to 50

A

8 to 20

How well did you know this?
1
Not at all
2
3
4
5
Perfectly
176
Q

If an average of 10 platelets are seen per oil immersion field, what is the estimated platelet count?

50 x 10 9th/L
100 x 10 9th/L
200 x 10 9th/L
300 x 10 9th/L

A

200 x 10 9th/L

For platelet estimate on a wedge smear:
Factor is 20,000

How well did you know this?
1
Not at all
2
3
4
5
Perfectly
177
Q

In the Ivy method of bleeding time, the blood pressure cuff is inflated to:

20 mm. Hg
30 mm. Hg
40 mm. Hg
45 mm. Hg

A

40 mm. Hg

How well did you know this?
1
Not at all
2
3
4
5
Perfectly
178
Q

Normal platelet adhesion depends upon:

Fibrinogen
Glycoprotein Ib
Glycoprotein IIb, IIIa complex
Calcium

A

Glycoprotein Ib

Glycoprotein Ib is a platelet receptor for VWF. Glycoprotein Ib and VWF are both necessary
for a normal platelet adhesion. Other proteins that play a role in platelet adhesion are
glycoproteins V and IX

How well did you know this?
1
Not at all
2
3
4
5
Perfectly
179
Q

Storage pool deficiencies are defects of:

Platelet adhesion
Platelet aggregation
Platelet granules
Platelet production

A

Platelet granules

Storage pool deficiencies are defects of platelet granules. Most commonly, a decrease in
platelet-dense granules is present with decreased release of ADP, ATP, calcium, and
serotonin from platelet-dense granules.

How well did you know this?
1
Not at all
2
3
4
5
Perfectly
180
Q

Which defect characterizes Gray’s syndrome?

Platelet adhesion defect
Dense granule defect
Alpha granule defect
Coagulation defect

A

Alpha granule defect

Gray’s syndrome is a platelet granule defect associated with a decrease in alpha granules
resulting in decreased production of alpha granule proteins such as platelet factor 4 and
beta thromboglobulin. Alpha granule deficiency results in the appearance of agranular
platelets when viewed on a Wright’s stained blood smear.

How well did you know this?
1
Not at all
2
3
4
5
Perfectly
181
Q

Hereditary hemorrhagic telangiectasia is a disorder of:

Platelets
Clotting proteins
Fibrinolysis
Connective tissue

A

Connective tissue

Hereditary hemorrhagic telangiectasia (Osler-Weber-Rendu syndrome) is a connective
tissue disorder associated with telangiectases (dilated capillaries) of the mucous
membranes and skin. Lesions may develop on the tongue, lips, palate, face, hands, nasal
mucosa, and throughout the gastrointestinal tract. This disorder is an autosomal dominant
condition that usually manifests in adolescence or early adulthood.

How well did you know this?
1
Not at all
2
3
4
5
Perfectly
182
Q

In which of the following lists the steps of hemostatic response in the correct order?

Fibrinolysis → injury → secondary hemostasis → primary hemostasis
Injury → primary hemostasis → secondary hemostasis → fibrinolysis
Injury → secondary hemostasis → primary hemostasis → fibrinolysis
Injury → fibrinolysis → primary hemostasis → secondary hemostasis

A

Injury → primary hemostasis → secondary hemostasis → fibrinolysis

How well did you know this?
1
Not at all
2
3
4
5
Perfectly
183
Q

Primary substrate of thrombin:

Fibrinogen
Prothrombin
Factor V
Factor X

A

Fibrinogen

How well did you know this?
1
Not at all
2
3
4
5
Perfectly
184
Q

Which results are associated with hemophilia A?

Prolonged APTT, normal PT
Prolonged PT and APTT
Prolonged PT, normal APTT
Normal PT and APTT

A

Prolonged APTT, normal PT

Hemophilia A is associated with factor VIII deficiency. Factor VIII is a factor in the intrinsic coagulation pathway that is evaluated by the APTT and not the PT test. The PT test evaluates the extrinsic and common pathways.

How well did you know this?
1
Not at all
2
3
4
5
Perfectly
185
Q

Normal PT and APTT results in a patient with a poor wound healing may be associated with:

Factor VII deficiency
Factor VIII deficiency
Factor XII deficiency
Factor XIII deficiency

A

Factor XIII deficiency

Factor XIII deficiency can lead to impaired wound healing and may cause severe bleeding problems. Factor XIII is a fibrin stabilizing factor that changes the fibrinogen bonds in fibrin polymers to stable covalent bonds. Factor XIII is not involved in the process of fibrin formation and, therefore, the PT and APTT are both normal.

How well did you know this?
1
Not at all
2
3
4
5
Perfectly
186
Q

Which coagulation factor is present in the highest concentration in plasma?

Factor II
Factor XII
Factor I
Factor VII

A

Factor I

How well did you know this?
1
Not at all
2
3
4
5
Perfectly
187
Q

Which of the following participates ONLY in the extrinsic pathway?

Factor VII
Factor IX
Factor X
Factor II

A

Factor VII

How well did you know this?
1
Not at all
2
3
4
5
Perfectly
188
Q

Plasma thromboplastin or prothrombinase includes:

-Calcium ion only
-Complex of calcium ions and activated factor XI
-Complex of activated factor VII and calcium ions
-Complex of activated factors X and V, platelet factor 3 and calcium ions

A

Complex of activated factors X and V, platelet factor 3 and calcium ions

How well did you know this?
1
Not at all
2
3
4
5
Perfectly
189
Q

The activated partial thromboplastin time is NOT affected by deficiency of:

Factor VIII
Factor IX
Factor XI
Factor VII

A

Factor VII

How well did you know this?
1
Not at all
2
3
4
5
Perfectly
190
Q

Prothrombin time is NOT affected by a deficiency of:

Factor VIII
Factor V
Factor I
Factor X

A

Factor VIII

How well did you know this?
1
Not at all
2
3
4
5
Perfectly
191
Q

Classic hemophilia is a condition in which there may be a:

Prolonged bleeding time
Decrease in platelets
Prolonged prothrombin time
Prolonged activated partial thromboplastin time

A

Prolonged activated partial thromboplastin time

How well did you know this?
1
Not at all
2
3
4
5
Perfectly
192
Q

Which of the following is vitamin K dependent?

Factor XII
Fibrinogen
Antithrombin III
Factor VII

A

Factor VII

How well did you know this?
1
Not at all
2
3
4
5
Perfectly
193
Q

Last factor to be depressed n vitamin K deficiency:

Factor II
Factor VII
Factor X
Factor IX

A

Factor II

How well did you know this?
1
Not at all
2
3
4
5
Perfectly
194
Q

Which of the following factors is not present in BaSO4 adsorbed plasma?

Factor VIII
Factor II
Factor XII
Factor V

A

Factor II

How well did you know this?
1
Not at all
2
3
4
5
Perfectly
195
Q

Which one of the following factors typically shows an increase in liver disease?

Factor VII
Factor VIII
Factor IX
Factor X

A

Factor VIII

How well did you know this?
1
Not at all
2
3
4
5
Perfectly
196
Q

Which of the following factor deficiencies is associated with either no bleeding or only a minor bleeding tendency, even after trauma or surgery?

Factor X
Factor XII
Factor XIII
Factor V

A

Factor XII

How well did you know this?
1
Not at all
2
3
4
5
Perfectly
197
Q

In which of the following diseases would you most likely find an abnormal prothrombin time:

Hemophilia A
Hemophilia B
vWD
DIC

A

DIC

How well did you know this?
1
Not at all
2
3
4
5
Perfectly
198
Q

Increased APTT with a normal PT would indicate a deficiency of:

Factor II
Factor VII
Factor IX
Factor X

A

Factor IX

How well did you know this?
1
Not at all
2
3
4
5
Perfectly
199
Q

Normal APTT with an increased PT would indicate a deficiency of:

Factor II
Factor VII
Factor I
Factor IX

A

Factor VII

How well did you know this?
1
Not at all
2
3
4
5
Perfectly
200
Q

Increased APTT and PT would indicate a deficiency of:

Factor V
Factor XI
Factor XII
Factor VIII

A

Factor V

How well did you know this?
1
Not at all
2
3
4
5
Perfectly
201
Q

PTT measures all factors except for:

I and V
VIII and IX
V and VIII
VII and XIII

A

VII and XIII

How well did you know this?
1
Not at all
2
3
4
5
Perfectly
202
Q

A patient on therapeutic warfarin will most likely have a(an):

-Normal PT/INR, increased APTT, prolonged bleeding time, low platelet count
-Increased PT/INR, increased APTT, normal bleeding time, normal platelet count
-Normal PT/INR, normal APTT, normal bleeding time, normal platelet count
-Increased PT/INR, normal APTT, prolonged bleeding time, low platelet count

A

Increased PT/INR, increased APTT, normal bleeding time, normal platelet count

203
Q

Reversal of heparin overdose can be achieved by administration of:

Vitamin K
Anti-thrombin
Protamine sulfate
Warfarin

A

Protamine sulfate

204
Q

The preferred blood product for a bleeding patient with von Willebrand’s disease is transfusion with:

Factor II, VII, IX, X concentrates
Platelet Concentrates
Fresh Frozen Plasma and Platelets
Cryoprecipitated AHF

A

Cryoprecipitated AHF

205
Q

Which of the following laboratory findings is associated with Factor XIII deficiency?

Prolonged activated partial thromboplastin time
Clot solubility in a 5 molar urea solution
Prolonged thrombin time
Prolonged prothrombin time

A

Clot solubility in a 5 molar urea solution

206
Q

The following results were obtained on a patient: prolonged bleeding time, normal platelet count, normal PT, and prolonged APTT. Which of the following disorders is most consistent with these results?

Hemophilia A
Hemophilia B
vWD
Glanzmann’s thrombasthenia

A

vWD

207
Q

A patient has a history of mild hemorrhagic episodes. Laboratory results include a prolonged prothrombin time and activated partial thromboplastin time. The abnormal prothrombin time was corrected by normal and adsorbed plasma, but not aged serum. Which of the following coagulation factors is deficient?

Prothrombin
Factor V
Factor X
Factor VII

A

Factor V

208
Q

A 56-year-old woman was admitted to the hospital with a history of a moderate to severe BLEEDING tendency of a several years’ duration. Epistaxis and menorrhagia were reported. Prolonged APTT was corrected with fresh normal plasma, adsorbed plasma, and aged serum. Deficiency of which of the following is most likely?

Factor XII
Factor VIII
Factor XI
Factor IX

A

Factor XI

Prolonged APTT corrected with normal plasma, adsorbed plasma and aged serum:
Deficiency of factor XI or XII
Factor XI deficiency - patient exhibits bleeding (hemophilia C)
Factor XII deficiency - negative bleeding

209
Q

The abnormal APTT seen in pathological circulating anticoagulant is:

Corrected with aged serum
Corrected with adsorbed plasma
Corrected with normal plasma
Not corrected with any of the above

A

Not corrected with any of the above

210
Q

The activity of the lupus anticoagulant and anticardiolipin antibodies appears to be directed against:

Factor V
Factor VIII
Factor IX
Phospholipid

A

Phospholipid

211
Q

A prolonged Stypven (Russell viper venom) time is associated with deficiency of the following factors EXCEPT:

Factor I
Factor II
Factor X
Factor VII

A

Factor VII

212
Q

The laboratory test for monitoring heparin therapy is:

PT
PTT
Bleeding time
Thrombin time

A

PTT

213
Q

All of the following tests are affected by heparin therapy except:

Thrombin time
Whole blood clotting time
APTT
Reptilase time

A

Reptilase time

214
Q

An abnormal thrombin time is associated with:

Factor X deficiency
Excess plasminogen
Fibrinogen deficiency
Protein C deficiency

A

Fibrinogen deficiency

215
Q

The observation of a normal reptilase time and a prolonged thrombin time is indicative of:

Presence of fibrin degradation products
Hypoplasminogenemia
Dysfibrinogenemia
Presence of heparin

A

Presence of heparin

216
Q

Which of the following coagulation test results is normal in patient with classic vWD?

Bleeding time
APTT
Platelet count
Factor VIII:C and vWF

A

Platelet count

217
Q

Primary inhibitor of the fibrinolytic system?

Protein C
Protein S
Alpha2 antiplasmin
Alpha2 macroglobulin

A

Alpha2 antiplasmin

218
Q

The D-dimer test is a specific test for:

Plasminogen activation
Plasmin degradation of fibrinogen
Plasmin degradation of fibrin
Factor XIII

A

Plasmin degradation of fibrin

219
Q

Acute disseminated intravascular coagulation is characterized by:

Hypofibrinogenemia
Thrombocytosis
Negative D-dimer
Shortened thrombin time

A

Hypofibrinogenemia

220
Q

A positive protamine sulfate is suggestive of:

vWD
Primary fibrinolysis
DIC
Glanzmann’s thrombasthenia

A

DIC

221
Q

Which factor deficiency is associated with a prolonged PT and APTT?

X
VIII
IX
XI

A

X

Factor X, a common pathway factor deficiency, is most likely suspected, because both PT and APTT are prolonged. Other causes may include liver disease, vitamin K deficiency, and anticoagulant drugs such as Coumadin and heparin.

222
Q

The following results were obtained on a patient: normal platelet count and function, normal PT, and prolonged APTT. Which of the following disorders is most consistent with these results?

Hemophilia A
Bernard-Soulier syndrome
von Willebrand’s disease
Glanzmann’s thrombasthenia

A

Hemophilia A

Hemophilia A is associated with the deficiency of factor VIII resulting in bleeding and an abnormal APTT. The platelet number and function are normal in this disorder.

Von Willebrand’s disease is a disorder of platelet adhesion associated with decreased VWF and factor VIII, causing an abnormal platelet function test and an abnormal APTT test.

Both Glanzmann’s thrombasthenia and Bernard-Soulier syndrome cause deficient platelet aggregation, but do not cause an abnormal APTT.

223
Q

Fibrin monomers are increased in which of the following conditions?

Primary fibrinolysis
DIC
Factor VIII deficiency
Fibrinogen deficiency

A

DIC

Increased fibrin monomers result from coagulation activation. DIC is an acquired condition associated with spontaneous activation of coagulation and fibrinolysis.
In primary fibrinolysis, the fibrinolytic system is activated and fibrin monomers are normal.

224
Q

Which of the following is associated with multiple factor deficiencies?

An inherited disorder of coagulation
Severe liver disease
Dysfibrinogenemia
Lupus anticoagulant

A

Severe liver disease

Most of the clotting factors are made in the liver. Therefore, severe liver disease results in multiple factor deficiencies.

225
Q

Which of the following is an appropriate screening test for the diagnosis of lupus anticoagulant?

Thrombin time test
Diluted Russell’s viper venom test (DRVVT)
D-dimer test
FDP test

A

Diluted Russell’s viper venom test (DRVVT)

Russell’s viper venom (RVV) reagent contains factors X and V, activating enzymes that are strongly phospholipid dependent. The reagent also contains RVV, calcium ions, and phospholipid. In the presence of phospholipid autoantibodies such as lupus anticoagulant, the reagent phospholipid is partially neutralized causing prolongation of the clotting time

226
Q

What clotting factors (cofactors) are inhibited by protein S?

V and X
Va and VIIIa
VIII and IX
VIII and X

A

Va and VIIIa

Factors Va and VIIIa are deactivated by protein S and activated protein C.

227
Q

The Bethesda assay is used for which determination?

Lupus anticoagulant titer
Factor VIII inhibitor titer
Factor V Leiden titer
Protein S deficiency

A

Factor VIII inhibitor titer

The Bethesda assay is a quantitative assay for factor VIII inhibitor. In this assay, normal plasma is incubated with different dilutions of the patient’s plasma or a normal control. The inhibitor inactivates factor VIII present in normal plasma following incubation for 2 hours at 37°C. The residual activities in the sample are determined, and the inhibitor titer is calculated.

228
Q

Fibrinogen, which has been implicated as a primary risk factor for thrombotic disorders, increases approximately _____ mg/dL per DECADE in the elderly (65 to 79 years), 174 from 280 mg/dL to over 300 mg/dL.

1 mg/dL per decade
5 mg/dL per decade
10 mg/dL per decade
15 mg/dL per decade

A

10 mg/dL per decade

229
Q

The only abnormal test result in CHRONIC DIC:

PT
APTT
Thrombin time
D-dimer

A

D-dimer

DIC, although characteristically identified through its hemorrhagic symptoms, is classified as a THROMBOTIC DISORDER

  1. ACUTE DIC UNCOMPENSATED
    PT, PTT, and thrombin time are prolonged; the fibrinogen level is reduced to less than 100 mg/dL; and fibrin degradation products, including D-dimers, are significantly increased
  2. CHRONIC DIC COMPENSATED
    Only elevated test result may be the D-dimer assay value, a hallmark of unregulated coagulation and fibrinolysis
230
Q

DIC is also known as:

Defibrination syndrome
Consumption coagulopathy
Both of these
None of these

A

Both of these

DIC involves all hemostatic systems: vascular intima, platelets, leukocytes, coagulation, coagulation control pathways, and fibrinolysis.

231
Q

Assess deficiencies of factors II, V, VII, or X:

Platelet count
Prothrombin time (PT)
Partial thromboplastin time (PTT)
Thrombin time

A

Prothrombin time (PT)

SCREENING TESTS FOR GENERALIZED HEMOSTATIC DISORDER
1. HEMOGLOBIN, HEMATOCRIT; RETICULOCYTE COUNT:
Anemia associated with chronic bleeding; bone marrow response
2. PLATELET COUNT: Thrombocytopenia
3. PT: Deficiencies of factors II (prothrombin), V, VII, or X
4. PTT: Deficiencies of all factors except VII and XIII
5: THROMBIN TIME OR FIBRINOGEN ASSAY:
Hypofibrinogenemia and dysfibrinogenemia

232
Q

Assess deficiencies of all factors except VII and XIII:

Platelet count
Prothrombin time (PT)
Partial thromboplastin time (PTT)
Thrombin time

A

Partial thromboplastin time (PTT)

SCREENING TESTS FOR GENERALIZED HEMOSTATIC DISORDER

  1. HEMOGLOBIN, HEMATOCRIT; RETICULOCYTE COUNT:
    Anemia associated with chronic bleeding; bone marrow response
  2. PLATELET COUNT: Thrombocytopenia
  3. PT: Deficiencies of factors II (prothrombin), V, VII, or X
  4. PTT: Deficiencies of all factors except VII and XIII
    5: THROMBIN TIME OR FIBRINOGEN ASSAY:
    Hypofibrinogenemia and dysfibrinogenemia
233
Q

The target INR for PULMONARY EMBOLISM (PE) treatment:

1
2
3
4

A

3

INR 2-3:
Recommended for most indications (e.g., treatment or prophylaxis of deep venous thrombosis [DVT], or prevention of further clotting in patients who have had a myocardial infarction)
INR 2.5 - 3.5:
Recommended for patients with prosthetic heart valves
INR 3:
Pulmonary embolism (PE) treatment

234
Q

Which of the following is considered to be an advantage of the MECHANICAL end-point detection methodology?

It is not affected by lipemia in the test sample
It has the ability to provide a graph of clot formation
It can incorporate multiple wavelengths into a single testing sequence
It can measure proteins that do not have fibrin formation as the end-point

A

It is not affected by lipemia in the test sample

A hemostasis laboratory manager may choose to maintain a separate mechanical end-point coagulometer to substitute for the optical instrument if the specimen is too cloudy for optical determinations.

235
Q

In end-stage liver disease, the fibrinogen level may fall to less than ___mg/dL, which is a mark of liver failure.

Less than 100 mg/dL
Less than 200 mg/dL
Less than 300 mg/dL
Less than 400 mg/dL

A

Less than 100 mg/dL

In end-stage liver disease, the fibrinogen level may fall to less than 100 mg/dL, which is a mark of liver failure.

236
Q

PT and APTT are prolonged when the fibrinogen level is ____ mg/dL or less.

100 mg/dL or less
130 mg/dL or less
150 mg/dL or less
200 mg/dL or less

A

100 mg/dL or less

237
Q

Which of the following locations is not a site of extramedullary hematopoiesis?

Bone marrow
Liver
Spleen
Thymus

A

Bone marrow

BONE MARROW - MEDULLARY SITE

238
Q

Which of the following is decreased in cases of intravascular hemolytic anemia?

Bilirubin
Urine hemosiderin
Haptoglobin
Serum hemoglobin

A

Haptoglobin

239
Q

Patients with renal failure often exhibit compromised hematopoietic activity because of which of the following?

Concurrent depression of the thyroid
Decreased production of erythropoietin
Decreased production of GM-CSF
BM suppression caused by medications

A

Decreased production of erythropoietin

240
Q

Which laboratory test is best used for DEFINITIVE diagnosis of sickle cell anemia?

Solubility testing
Hemoglobin electrophoresis
Peripheral smear review for sickle cells
Bone marrow analysis

A

Hemoglobin electrophoresis

241
Q

Which of the following best describes the function of the Rapoport-Leubering pathway?

It produces ATP to help maintain RBC membrane deformability
It results in reduction of glutathione
It produces 2,3 diphosphoglycerate (2,3 DPG)
It produces cytochrome reductase

A

It produces 2,3 diphosphoglycerate (2,3 DPG)

242
Q

The degree of effective erythropoiesis is best assessed by:

Serum iron levels
Serial hemoglobin determinations
Reticulocyte count
Ferrokinetic studies

A

Reticulocyte count

243
Q

Which of the following hemoglobins is composed of four beta globin chains?

Hemoglobin F
Hemoglobin C
Bart’s hemoglobin
Hemoglobin H

A

Hemoglobin H

244
Q

Which of the following cells exhibit IgE receptors on their surface membranes?

Basophils
Eosinophils
Band neutrophils
Monocytes

A

Basophils

245
Q

Which cells are involved in immediate hypersensitivity reactions?

Eosinophils
Basophils
Plasma cells
Reactive lymphocytes

A

Basophils

246
Q

Which conditions which shift the oxyhemoglobin dissociation curve to the right?

Acidosis
Alkalosis
Multiple blood transfusions
Increased quantities of hemoglobin S or C

A

Acidosis

247
Q

In which stage of erythrocytic maturation does hemoglobin formation begin?

Reticulocyte
Pronormoblast
Basophilic normoblast
Polychromatophilic normoblast

A

Polychromatophilic normoblast

248
Q

What is the last nucleated stage in development of erythrocyte?

Prorubricyte
Rubricyte
Metarubricyte
Reticulocyte

A

Metarubricyte

249
Q

Which is the major hemoglobin found in the RBCs of patients with SICKLE CELL TRAIT?

Hemoglobin S
Hemoglobin F
Hemoglobin A2
Hemoglobin A1

A

Hemoglobin A1

250
Q

All of the following are associated with hemolytic anemia except:

Methemoglobinemia
Hemoglobinuria
Hemoglobinemia
Increased haptoglobin

A

Increased haptoglobin

Decreased haptoglobin

251
Q

Which of these hemoglobin derivatives cannot be reduced back to normal hemoglobin?

Methemoglobin
Hemiglobin
Carboxyhemoglobin
Sulfhemoglobin

A

Sulfhemoglobin

252
Q

Which of the following is characteristic of cellular changes as megakaryoblasts mature into megakaryocytes within the bone marrow?

Progressive decrease in overall cell size
Increasing basophilia of cytoplasm
Nuclear division without cytoplasmic division
Fusion of the nuclear lobes

A

Nuclear division without cytoplasmic division

253
Q

he type of nuclear reproduction seen in megakaryocytes is:

Polypoid mitosis
Endomitosis
Meiosis
Binary fission

A

Endomitosis

254
Q

The outermost zone of platelet is called:

Peripheral zone
Sol-gel zone
Alpha zone
Organelle zone

A

Peripheral zone

255
Q

Which of the following cells is the largest cell in the BONE MARROW?

Monocyte
Megakaryocyte
Osteoblast
Mast cell

A

Megakaryocyte

256
Q

The number of platelets an average megakaryocyte generates is approximately:

25 to 50
50 to 200
200 to 500
2,000 to 4,000

A

2,000 to 4,000

257
Q

Portion of DNA that is ACTIVE in gene expression and stains lightly with Wright stain:

Euchromatin
Heterochromatin

A

Euchromatin

258
Q

Portion of DNA that is INACTIVE during transcription to messenger RNA and stains deeply with Wright stain:

Euchromatin
Heterochromatin

A

Heterochromatin

259
Q

Apoptosis is morphologically identified by:

Cellular swelling
Nuclear condensation
Rupture of the cytoplasm
Rupture of the nucleus

A

Nuclear condensation

APOPTOSIS
CELL SIZE: reduced due to shrinkage
NUCLEUS: condensation and fragmentation between nucleosomes
NECROSIS
CELL SIZE: enlarged due to swelling
NUCLEUS: random breaks and lysis (karylolysis)

260
Q

The morphologic manifestation of APOPTOSIS is _______ of the cell.

Shrinkage of the cell
Swelling of the cell

A

Shrinkage of the cell

261
Q

The first morphologic manifestation of NECROSIS is _______ of the cell.

Shrinkage of the cell
Swelling of the cell

A

Swelling of the cell

262
Q

Dendritic cells are derived from:

Common lymphoid progenitor
Common myeloid progenitor

A

Common lymphoid progenitor

263
Q

Which of the following red blood cell precursors is the LAST STAGE TO UNDERGO MITOSIS?

Pronormoblast
Basophilic normoblast
Polychromatophilic normoblast
Orthochromic normoblast

A

Polychromatophilic normoblast

264
Q

Tertiary granules of the neutrophils are formed during the: *

Promyelocyte stage
Myelocyte and metamyelocyte stage
Metamyelocyte and band stage
Band and segmented neutrophil stage

A

Metamyelocyte and band stage

Primary (Azurophilic) Granules
Formed during the promyelocyte stage
Last to be released (exocytosis)

Secondary (Specific) Granules
Formed during myelocyte and metamyelocyte stages
Third to be released

Tertiary Granules
Formed during metamyelocyte and band stages
Second to be released

Secretory Granules (Secretory Vesicles)
Formed during band and segmented neutrophil stages
First to be released (fuse to plasma membrane)

265
Q

Chondroitin sulfates such as heparan:

Neutrophil secondary granules
Eosinophil secondary granules
Basophil secondary granules

A

Basophil secondary granules

266
Q

Which erythrocyte metabolic pathway generates adenosine triphosphate (ATP) via glycolysis?

Embden-Meyerhof
Hexose monophosphate
Rapoport-Luebering
Methemoglobin reductase

A

Embden-Meyerhof

267
Q

ron is TRANSPORTED IN PLASMA via:

Hemosiderin
Ferritin
Transferrin
Hemoglobin

A

Transferrin

268
Q

Multilobed nucleus:

MK-I
MK-II
MK-III

A

MK-III

269
Q

Moderately condensed chromatin:

MK-I
MK-II
MK-III

A

MK-II

270
Q

Deeply and variably condensed chromatin:

MK-I
MK-II
MK-III

A

MK-III

271
Q

Reagent for the solubility test:

Ferric cyanide and detergent
2% sodium metabisulfite
Sodium hydrosulfite (dithionite) and a detergent-based lysing agent (saponin)
1% sodium hydroxide

A

Sodium hydrosulfite (dithionite) and a detergent-based lysing agent (saponin)

272
Q

Hemoglobin SOLUBILITY TEST is a screening test for:

Hemoglobin A2
Hemoglobin F
Hemoglobin S
Unstable hemoglobins

A

Hemoglobin S

273
Q

Color of blood in sulfhemoglobinemia:

Chocolate brown
Cherry red
Mauve lavender
Bright red

A

Mauve lavender

274
Q

The red blood cell protein that is responsible for deformability and flexibility of the red blood cell is:

Spectrin
Glycophorin
Glycine
Erythropoietin

A

Spectrin

275
Q

In hemoglobin C, glutamic acid on the 6th position of beta chain is replaced by which amino acid?

Lysine
Valine
Arginine
Glutamine

A

Lysine

276
Q

The most mature cell that can undergo mitosis is the:

Promyelocyte
Myelocyte
Metamyelocyte
Band

A

Myelocyte

277
Q

Granulocytic precursor with an indented or kidney-shaped nucleus:

Promyelocyte
Myelocyte
Metamyelocyte
Band

A

Metamyelocyte

278
Q

Granulocytic cell with a sausage-shaped nucleus:

Promyelocyte
Myelocyte
Metamyelocyte
Band

A

Band

279
Q

Most reliable way to differentiate a mature from an immature granulocyte:

Size of the cell
Color of the cytoplasm
Size of the nucleus
Chromatin pattern

A

Chromatin pattern

280
Q

Reliable way to differentiate PLATELET PRECURSORS:

Size of the cell
Cytoplasmic appearance
Nucleus
Chromatin pattern

A

Cytoplasmic appearance

281
Q

Alpha and dense granules of platelets:

In peripheral zone
In sol-gel zone
In organelle zone
In membranous system

A

In organelle zone

282
Q

The term for cell movement across the cellular membranes from the blood vessels to the tissues is called:

Diapedesis
Endocytosis
Margination
Chemotaxis

A

Diapedesis

283
Q

G1 or cell growth:

1 hour
4 hours
8 hours
10 hours

A

10 hours

CELL CYCLE. G0, nondividing cell; G1, cell growth (10 hours) ; S, DNA replication (8 hours); G2, protein synthesis (4 hours); M, mitosis (1 hour) followed by cytokinesis or cell division.

284
Q

S or DNA replication

1 hour
4 hours
8 hours
10 hours

A

8 hours

CELL CYCLE. G0, nondividing cell; G1, cell growth (10 hours) ; S, DNA replication (8 hours); G2, protein synthesis (4 hours); M, mitosis (1 hour) followed by cytokinesis or cell division.

285
Q

G2 or protein synthesis:

1 hour
4 hours
8 hours
10 hours

A

4 hours

CELL CYCLE. G0, nondividing cell; G1, cell growth (10 hours) ; S, DNA replication (8 hours); G2, protein synthesis (4 hours); M, mitosis (1 hour) followed by cytokinesis or cell division.

286
Q

M or mitosis: *

1 hour
4 hours
8 hours
10 hours

A

1 hour

287
Q

What is the ratio of IRON TO PYRROLE in 1 heme portion of the hemoglobin molecule?

1 iron to 2 pyrrole rings
2 iron to 1 pyrrole ring
1 iron to 4 pyrrole rings
4 iron to 1 pyrrole ring

A

1 iron to 4 pyrrole rings

The heme portion of the hemoglobin molecule consists of one iron (Fe2+) atom and four pyrrole rings that are joined to each other.
A complete hemoglobin molecule consists of four heme molecules, each of which is attached to one molecule of the protein globin.

288
Q

Densely packed chromatin:

Myelocyte
Metamyelocyte
Band
Segmented neutrophil

A

Segmented neutrophil

289
Q

Very clumped chromatin:

Myelocyte
Metamyelocyte
Band
Segmented neutrophil

A

Band

290
Q

Basophils have an average circulation time of about ____ hours.

7 hours
8.5 hours
9 hours
12 hours

A

8.5 hours

Basophils have an average circulation time of about 8.5 hour

291
Q

Basophils remain in the maturation-storage phase for:

7 hours
8.5 hours
9.5 hours
12 hours

A

12 hours

Basophils remain in this phase for the shortest period, approximately 12 hours.

292
Q

A patient has macrocytic anemia, the physician suspects pernicious anemia. Which tests would best rule in a definitive diagnosis of pernicious anemia?

Homocysteine
Intrinsic factor antibodies
Ova and parasite examination for D. latum
Bone marrow examination

A

Intrinsic factor antibodies

293
Q

G6PD deficiency episodes are related to which of the following?

Exposure to oxidant drugs
Defective globin chains
Antibodies to RBCs
Abnormal protein structures

A

Exposure to oxidant drugs

294
Q

Lack of vitamin B12 or folic acid hinders the erythroblast in manufacturing:

Heme
Globin
DNA
RNA

A

DNA

295
Q

The nitroblue tetrazolium reduction test is used to assist in the diagnosis of:

Leukocyte adhesion disorders (LADs)
Chronic granulomatous disease (CGD)
May-Hegglin anomaly
Pelger-Huet anomaly

A

Chronic granulomatous disease (CGD)

296
Q

Which of the following erythrocyte inclusions can be visualized with supravital stain but cannot be detected on a Wright’s-stained blood smear?

Basophilic stippling
Heinz bodies
Howell-Jolly bodies
Siderotic granules

A

Heinz bodies

297
Q

Which antibody is associated with paroxysmal cold hemoglobinuria (PCH)?

Anti-I
Anti-i
Anti-M
Anti-P

A

Anti-P

298
Q

Bite cells are usually seen in patients with:

Rh null disease
Chronic granulomatous disease
G6PD deficiency
Pyruvate kinase deficiency

A

G6PD deficiency

299
Q

Which antibiotic(s) is (are) most often implicated in the development of aplastic anemia?

Sulfonamides
Penicillin
Tetracycline
Chloramphenicol

A

Chloramphenicol

300
Q

Which anemia has red cell morphology similar to that seen in iron deficiency anemia?

Sickle cell anemia
Thalassemia
Pernicious anemia
Hereditary spherocytosis

A

Thalassemia

301
Q

Auer rods may be seen in all of the following except:

Acute myeloid leukemia
Acute promyelocytic leukemia
Acute lymphoblastic leukemia
Acute myelomonocytic leukemia

A

Acute lymphoblastic leukemia

302
Q

In myeloid cells, the stain that selectively identifies PHOSPHOLIPIDS in the membranes of both primary and secondary granules is:

PAS
Myeloperoxidase
Sudan black B
Tdt

A

Sudan black B

303
Q

Sodium fluoride maybe added to naphthyl ASD (NASDA) esterase reaction. The fluoride is added to inhibit a positive reaction with:

Megakaryocytes
Monocytes
Erythrocytes
Granulocytes

A

Monocytes

304
Q

What is the characteristic finding seen in the peripheral smear of a patient with multiple myeloma?

Microcytic hypochromic cells
Intracellular inclusion bodies
Rouleaux
Hypersegmented neutrophils

A

Rouleaux

305
Q

Of the following, the disease most closely associated with granulocyte hyposegmentation is:

May-Hegglin anomaly
Pelger-Huet anomaly
Chediak-Higashi syndrome
Gaucher’s disease

A

Pelger-Huet anomaly

306
Q

Which of the following is associated with Alder-Reilly inclusions?

Membrane defect of lysosomes
Dohle bodies and giant platelets
Two-lobed neutrophils
Mucopolysaccharidosis

A

Mucopolysaccharidosis

307
Q

Asynchronous development of hematopoietic cells within the bone marrow is the result of:

Inadequate levels of RNA
Decreased erythropoietin
Defective stem cells
Impaired DNA synthesis

A

Impaired DNA synthesis

308
Q

Megaloblastic anemia is characterized by all of the following, EXCEPT:

Decreased WBCs and retics
Hypersegmented neutrophils
Oval macrocytes
Increased platelets

A

Increased platelets

309
Q

Many microspherocytes and schistocytes and budding off of spherocytes can be seen on peripheral blood smears of patients with:

Hereditary spherocytosis
Disseminated intravascular coagulation
Autoimmune hemolytic anemia
Extensive burns

A

Extensive burns

310
Q

In myelofibrosis, the characteristic abnormal red cell morphology is:

Target cells
Schistocytes
Teardrop cells
Ovalocytes

A

Teardrop cells

311
Q

In the French-American-Bristish (FAB) classification, myelomonocytic leukemia would be:

M1 and M2
M3
M4
M5

A

M4

312
Q

A cytogenic abnormality is found in almost 50% of the patients with which of the following classifications of acute nonlymphocytic leukemia?

M1
M3
M5
M6

A

M3

313
Q

DIC is most often associated with which of the following FAB designation of acute leukemia?

M1
M3
M4
M5

A

M3

314
Q

Which of the following stains is helpful in the diagnosis of suspected erythroleukemia?

Peroxidase
Periodic acid-Schiff
Nonspecific esterase
Acid phosphatase

A

Periodic acid-Schiff

315
Q

Which type of anemia is usually present in a patient with acute leukemia?

Microcytic, hyperchromic
Microcytic, hypochromic
Normocytic, normochromic
Macrocytic, normochromic

A

Normocytic, normochromic

316
Q

In the French-American-British (FAB) classification, acute lymphocytic leukemia is divided into groups according to:

Prognosis
Immunology
Cytochemistry
Morphology

A

Morphology

317
Q

The FAB classification of ALL seen most commonly in children is:

L1
L2
L3
Burkitt’s type

A

L1

318
Q

In addition to the number of blasts, what other criterion is essential for the diagnosis of RARS?

More than 15% ringed sideroblasts
Dyshematopoiesis in all three lineages
More than 5% ringed sideroblasts
Pancytopenia

A

More than 15% ringed sideroblasts

319
Q

Using an electronic cell counter analyzer, an increased RDW should correlate with:

Spherocytosis
Anisocytosis
Leukocytosis
Presence of NRBCs

A

Anisocytosis

The RDW parameter correlates with the degree of anisocytosis seen on the morphological examination. The reference range is 11.5%-14.5%

320
Q

Which of the following organs is responsible for the “pitting process” for RBCs?

Liver
Spleen
Kidney
Lymph nodes

A

Spleen

The spleen is the supreme filter of the body, pitting imperfections from the erythrocyte without destroying the integrity of the membrane.

321
Q

Which of the following conditions may produce spherocytes in a peripheral smear?

Pelger-Huët anomaly
Pernicious anemia
Autoimmune hemolytic anemia
Sideroblastic anemia

A

Autoimmune hemolytic anemia

Spherocytes are produced in autoimmune hemolytic anemia.

322
Q

Iron deficiency anemia may be distinguished from anemia of chronic infection by:

Serum iron level
Red cell morphology
Red cell indices
Total iron-binding capacity

A

Total iron-binding capacity

In iron deficiency anemia, the serum iron and ferritin levels are decreased and the total iron-binding capacity and RBC protoporphyrin are increased.

In chronic disease, serum iron and TIBC are both decreased because the iron is trapped in reticuloendothelial (RE) cells, and is unavailable to the
red cells for hemoglobin production.

323
Q

Storage iron is usually best determined by:

Serum transferrin levels
Hgb values
Myoglobin values
Serum ferritin levels

A

Serum ferritin levels

Ferritin enters the serum from all ferritin-producing tissues, and therefore is considered to be a good indicator of body storage iron.

Because iron stores must be depleted before anemia develops, low serum ferritin levels precede the fall in serum iron associated with iron deficiency anemia.

324
Q

Which morphological classification is characteristic of megaloblastic anemia?

Normocytic, normochromic
Microcytic, normochromic
Macrocytic, hypochromic
Macrocytic, normochromic

A

Macrocytic, normochromic

325
Q

Which anemia is characterized by a lack of intrinsic factor that prevents B12 absorption?

Tropical sprue
Transcobalamin deficiency
Blind loop syndrome
Pernicious anemia

A

Pernicious anemia

Pernicious anemia is caused by a lack of intrinsic factor, which prevents vitamin B12 absorption.

326
Q

The macrocytes typically seen in megaloblastic processes are:

Crescent-shaped
Teardrop-shaped
Ovalocytic
Pencil-shaped

A

Ovalocytic

Macrocytes in true megaloblastic conditions are oval macrocytes as opposed to the round macrocytes that are usually seen in alcoholism and obstructive liver disease.

327
Q

The morphological characteristic(s) associated with the Chédiak-Higashi syndrome is (are):

Pale blue cytoplasmic inclusions
Giant lysosomal granules
Small, dark-staining granules and condensed nuclei
Nuclear hyposegmentation

A

Giant lysosomal granules

Chédiak-Higashi syndrome is a disorder of neutrophil phagocytic dysfunction caused by depressed chemotaxis and delayed degranulation. The degranulation disturbance is attributed to interference from the giant lysosomal granules characteristic of this disorder.

328
Q

The WHO classification requires what percentage for the blast count in the blood or bone marrow for the diagnosis of AML?

At least 30%
At least 20%
At least 10%
Any percentage

A

At least 20%

329
Q

Howell-Jolly bodies are composed of:

DNA
Iron
Reticulum
RNA

A

DNA

330
Q

When acanthocytes are found on the blood smear, it is usually the result of

Abnormal membrane permeability
Altered membrane lipids
Mechanical trauma
Polymerization of hemoglobin molecules

A

Altered membrane lipids

331
Q

The morphologic abnormality characteristically found in hemoglobinopathies:

Elliptocytes
Dacryocytes
Codocytes
Discocytes

A

Codocytes

332
Q

Hereditary stomatocytosis is manifested physiologically by changes in:

Hemoglobin oxygen affinity
Membrane cation permeability
Efficiency of hemoglobin reduction
Glycolytic ATP production

A

Membrane cation permeability

The major defect in hereditary stomatocytosis is altered permeability of the red cell membrane to Na+ and K+ ions.

333
Q

Abetalipoproteinemia is characterized by mild anemia and numerous on the peripheral blood smear.

Acanthocytes
Elliptocytes
Echinocytes
Stomatocytes

A

Acanthocytes

334
Q

The fish tapeworm Diphyllobothrium latum is associated with the development of:

Microcytic anemia
Macrocytic anemia
Hemolytic anemia
Hypoproliferative anemia

A

Macrocytic anemia

The fish tapeworm competes for vitamin B12, and a macrocytic (megaloblastic) anemia may develop.

Hookworm infestation causes chronic blood loss and a microcytic anemia due to iron deficiency.

335
Q

Which of the following represents the principal defect in chronic granulomatous disease (CGD)?

Chemotactic migration
Phagocytosis
Lysosomal formation and function
Oxidative respiratory burst

A

Oxidative respiratory burst

Chronic granulomatous disease (CGD) is a hereditary disorder in which neutrophils are incapable of killing most ingested microbes. The disease is usually fatal because of defective generation of oxidative metabolism products, such as superoxide anions and hydrogen peroxide, which are essential for killing.

336
Q

A Gaucher cell is best described as a macrophage with:

“Wrinkled” cytoplasm due to an accumulation of glucocerebroside
“Foamy” cytoplasm filled with unmetabolized sphingomyelin
Pronounced vacuolization and deposits of cholesterol
Abundant cytoplasm containing storage iron and cellular remnants

A

“Wrinkled” cytoplasm due to an accumulation of glucocerebroside

Gaucher disease is a lipid storage disorder in which there is an accumulation of glucocerebroside in the macrophages because of a genetic lack of glucocerebrosidase, an enzyme required for normal lipid metabolism. Gaucher cells are found in the liver, spleen, and bone marrow.

337
Q

The familial disorder featuring pseudo-Dohle bodies, thrombocytopenia, and large platelets is called:

May-Hegglin anomaly
Chediak-Higashi syndrome
Pelger-Huet anomaly
Alder-Reilly anomaly

A

May-Hegglin anomaly

May-Hegglin anomaly is an autosomal dominant disorder in which large blue cytoplasmic structures that resemble Dohle bodies are found in the granulocytes and possibly the monocytes.

338
Q

Perl’s Prussian blue is a stain used to detect:

DNA
RNA
Iron
Glycogen

A

Iron

When stained with a mixture of potassium ferricyanide and hydrochloric acid, nonheme iron stains bright blue. This is the most common stain used for storage iron. It can be used on bone marrow to identify sideroblasts, peripheral blood to identify the presence of siderocytes, or urine to perform hemosiderin testing.

339
Q

Solid tumor counterpart of acute lymphoblastic leukemia: *

Lymphoma, undifferentiated
Lymphoma, poorly-differentiated
Lymphoma, well-differentiated
Myeloma

A

Lymphoma, poorly-differentiated

LEUKEMIA TYPE SOLID TUMOR COUNTERPART

STEM CELL LEUKEMIA: Lymphoma, undifferentiated

ACUTE LYMPHOBLASTIC: Lymphoma, poorly leukemia differentiated; lymphocytic

CHRONIC LYMPHOCYTIC: Lymphoma, well leukemia differentiated; lymphocytic

MONOCYTIC LEUKEMIA: Reticulum cell sarcoma
ACUTE MYELOGENOUS: Chloroma granulocytic leukemia

PLASMA CELL LEUKEMIA: Myeloma

340
Q

Solid tumor counterpart of plasma cell leukemia: *

Lymphoma, undifferentiated
Reticulum cell sarcoma
Chloroma
Myeloma

A

Myeloma

LEUKEMIA TYPE SOLID TUMOR COUNTERPART

STEM CELL LEUKEMIA: Lymphoma, undifferentiated

ACUTE LYMPHOBLASTIC: Lymphoma, poorly leukemia differentiated; lymphocytic

CHRONIC LYMPHOCYTIC: Lymphoma, well leukemia differentiated; lymphocytic

MONOCYTIC LEUKEMIA: Reticulum cell sarcoma
ACUTE MYELOGENOUS: Chloroma granulocytic leukemia

PLASMA CELL LEUKEMIA: Myeloma

341
Q

FAB type M6 is characterized by increased:

Promyelocytes and lysozyme activity
Marrow megakaryocytes and thrombocytosis
Marrow erythroblasts and multinucleated red cells
Marrow monoblasts and immature monocytes

A

Marrow erythroblasts and multinucleated red cells

342
Q

It can be used to clean the objective lenses:

Normal saline
Formalin
Xylene
Lens cleaner or 70% isopropyl alcohol

A

Lens cleaner or 70% isopropyl alcohol

The use of xylene is discouraged, because it contains a carcinogenic component (benzene). Xylene is also a poor cleaning agent, leaving an oily film on the lens.

343
Q

All of the following are components of the modified Drabkin’s reagent EXCEPT:

Potassium cyanide, potassium ferricyanide
Dihydrogen potassium phosphate
Nonionic detergent
Sodium chloride

A

Sodium chloride

344
Q

Microhematocrit tube:

Length 75 mm, inner bore of 1.2 mm
Length 115 mm, bore of 3 mm
Length of 300 mm, bore of 2.5 mm
Length of 11.5 cm, bore of 3 mm

A

Length 75 mm, inner bore of 1.2 mm

345
Q

Duplicate hematocrit results should agree within __ unit (%). *

1%
2%
5%
15%

A

1%

346
Q

When comparing spun microhematocrit results with hematocrit results obtained on an electronic cell counter, the spun hematocrit results may vary from ___ because of this trapped plasma (unless the cell counter has been calibrated against spun microhematocrits uncorrected for trapped plasma).

1 to 3% lower
1 to 3% higher
3 to 5% lower
3 to 5% higher

A

1 to 3% higher

347
Q

The difference between the total cells counted on each side should be less than ___%. A greater variation could indicate an uneven distribution, which requires that the procedure be repeated.

Less than 10%
Less than 15%
Less than 20%
Less than 30%

A

Less than 10%

348
Q

If the WBC count is above 30 x 10 9th/L, it is advisable to use this dilution:

1:10
1:20
1:100
1:200

A

1:100

TYPICAL WBC DILUTION - 1:20
Above 30 x 10 9th/L - 1:100
100 to 300 x 10 9th/L - 1:200
Below 3 x 10 9th/L - 1:10

349
Q

For manual WBC count, the filled counting chamber should be allowed to stand for __ prior to performing the count to give the WBCs time to settle.

3 minutes
5 minutes
10 minutes
15 minutes

A

10 minutes

350
Q

For manual platelet count, the filled counting chamber should be allowed to stand for __ prior to performing the count to give the platelets time to settle.

3 minutes
5 minutes
10 minutes
15 minutes

A

15 minutes

351
Q

Insufficient centrifugation will result in:

A false increase in hematocrit (Hct) value
A false decrease in Hct value
No effect on Hct value
All of these options, depending on the patient

A

A false increase in hematocrit (Hct) value

Insufficient centrifugation does not pack down the red blood cells; therefore, the Hct, which is the volume of packed cells, will increase.

352
Q

What effect would using a buffer at pH 6.0 have on a Wright’s-stained smear?

Red cells would be stained too pink
White cell cytoplasm would be stained too blue
Red cells would be stained too blue
Red cells would lyse on the slide

A

Red cells would be stained too pink

The pH of the buffer is critical in Romanowsky stains. When the pH is too low (<6.4), the red cells take up more acid dye (eosin), becoming too pink. Leukocytes also show poor nuclear detail when the pH is decreased.

353
Q

A correction is necessary for WBC counts when nucleated RBCs are seen on the peripheral smear because:

The WBC count would be falsely lower
The RBC count is too low
Nucleated RBCs are counted as leukocytes
Nucleated RBCs are confused with giant platelets

A

Nucleated RBCs are counted as leukocytes

The automated hematology analyzers enumerate all nucleated cells. NRBCs are counted along with WBCs, falsely elevating the WBC count.

To correct the WBC count, determine the number of NRBCs per 100 WBCs. Corrected WBC count = (uncorrected WBC count ÷ [NRBC’s + 100]) × 100.

354
Q

In essential thrombocythemia, the platelets are:

Increased in number and functionally abnormal
Normal in number and functionally abnormal
Decreased in number and functional
Decreased in number and functionally abnormal

A

Increased in number and functionally abnormal

In essential thrombocythemia, the platelet count is extremely elevated. These platelets are abnormal in function, leading to both bleeding and thrombotic diathesis.

355
Q

Which of the following cells is considered pathognomonic for Hodgkin’s disease?

Niemann-Pick cells
Reactive lymphocytes
Flame cells
Reed-Sternberg cells

A

Reed-Sternberg cells

The morphological common denominator in Hodgkin’s lymphoma is the Reed-Sternberg (RS) cell. It is a large, binucleated cell with a dense nucleolus surrounded by clear space. These characteristics give the RS cell an “owl’s eye” appearance.

356
Q

In myelofibrosis, the characteristic abnormal red blood cell morphology is that of:

Target cells
Schistocytes
Teardrop cells
Ovalocytes

A

Teardrop cells

The marked amount of fibrosis, both medullary and extramedullary, accounts for the irreversible red cell morphological change to a teardrop shape. The red cells are “teared” as they attempt to pass through the fibrotic tissue.

357
Q

The erythrocytosis seen in relative polycythemia occurs because of:

Decreased arterial oxygen saturation
Decreased plasma volume of circulating blood
Increased erythropoietin levels
Increased erythropoiesis in the bone marrow

A

Decreased plasma volume of circulating blood

Relative polycythemia is caused by a reduction of plasma rather than an increase in red blood cell volume or mass. Red cell mass is increased in both PV and secondary polycythemia, but erythropoietin levels are high only in secondary polycythemia.

358
Q

Cells that exhibit a positive stain with acid phosphatase and are not inhibited with tartaric acid are characteristically seen in:

Infectious mononucleosis
Infectious lymphocytosis
Hairy cell leukemia
T-cell acute lymphoblastic leukemia

A

Hairy cell leukemia

359
Q

In addition to an increase in red blood cells, which of the following is characteristic of polycythemia vera?

-Decreased platelets, decreased granulocytes, decreased erythropoietin level
-Decreased platelets, decreased granulocytes, increased erythropoietin level
-Increased platelets, increased granulocytes, increased erythropoietin level
-Increased platelets, increased granulocytes, decreased erythropoietin level

A

Increased platelets, increased granulocytes, decreased erythropoietin level

PV is not due to the activity of erythropoietin. The production of erythropoietin is almost completely suppressed in this malignant condition.

360
Q

Which of the following is not a cause of ABSOLUTE secondary erythrocytosis?

Defective cardiac or pulmonary function
High-altitude adjustment
Dehydration secondary to diuretic use
Hemoglobins with increased oxygen affinity

A

Dehydration secondary to diuretic use

Dehydration is a cause of relative (pseudo) erythrocytosis due to plasma loss.

361
Q

Which of the following is NOT classified as a myeloproliferative disorder?

Polycythemia vera
Essential thrombocythemia
Multiple myeloma
Chronic myelogenous leukemia

A

Multiple myeloma

Multiple myeloma is a malignant lymphoproliferative disorder characterized by a clonal proliferation of plasma cells and multiple bone tumors.

362
Q

Which of the following gene mutations correlates with the t(9;22) that is present in Philadelphia chromosome positive chronic myelogenous leukemia?

MYC/IGH
BCR/ABL
PML/RARA
JAK2

A

BCR/ABL

The Philadelphia chromosome, t(9;22), is detected in almost all cases of CML (depends on detection method) and results in a mutated BCR/ABL fusion gene.

363
Q

In which of the following would an absolute monocytosis not be seen?

Tuberculosis
Recovery stage of acute bacterial infection
Collagen disorders
Infectious mononucleosis

A

Infectious mononucleosis

IM - LYMPHOCYTOSIS

364
Q

Naphthol AS-D chloroacetate esterase (specific) is usually positive in _____ cells, and alpha naphthyl acetate esterase (nonspecific)is useful for identifying blast cells of ______lineage.

Granulocytic; monocytic
Monocytic; granulocytic
Granulocytic; lymphocytic
Monocytic; lymphocytic

A

Granulocytic; monocytic

Naphthol AS-D chloroacetate esterase (specific) reacts strongly in granulocytic cells, and alpha-naphthyl acetate esterase (nonspecific) stains positively in monocytic cells.

365
Q

What is the key diagnostic test for Hodgkin lymphoma?

Bone marrow biopsy
Lymph node biopsy
Spinal tap
Skin biopsy

A

Lymph node biopsy

The test that would be the most beneficial for the diagnosis of Hodgkin lymphoma is a lymph node biopsy.

366
Q

Which of the following is NOT a characteristic finding in polycythemia vera?

Blood pancytosis
Increased red cell mass
Increased erythropoietin level
Increased blood viscosity

A

Increased erythropoietin level

Primary polycythemia (vera) is a malignant myeloproliferative disorder characterized by autonomous marrow production of erythrocytes in the presence of low erythropoietin levels.

367
Q

What combination of reagents is used to measure hemoglobin?

Hydrochloric acid and p-dimethylaminobenzaldehyde
Potassium ferricyanide and potassium cyanide
Sodium bisulfite and sodium metabisulfite
Sodium citrate and hydrogen peroxide

A

Potassium ferricyanide and potassium cyanide

The standard assay for hemoglobin utilizes potassium ferricyanide. This solution, formerly called Drabkin’s reagent, is now called cyanmethemoglobin (HiCN) reagent. The ferricyanide oxidizes hemoglobin iron from ferrous (Fe2+) to ferric (Fe3+), and the potassium cyanide stabilizes the pigment as cyanmethemoglobin for spectrophotometric measurement.

368
Q

The components of Wright’s stain include:

Crystal violet and safranin
Brilliant green and neutral red
New methylene blue and carbolfuchsin
Methylene blue and eosin

A

Methylene blue and eosin

One type of Romanowsky stain is the Wright’s stain. It is a polychrome stain consisting of methylene blue and eosin.

369
Q

Which of the following red cell inclusions stain with both Perl’s Prussian blue and Wright’s stain?

Howell-Jolly bodies
Basophilic stippling
Pappenheimer bodies
Heinz bodies

A

Pappenheimer bodies

Pappenheimer bodies are iron deposits associated with mitochondria, and they stain with both Perl’s Prussian blue and Wright’s stain.

370
Q

Which of the following is NOT a source of error when measuring hemoglobin by the cyanmethemoglobin method?

Excessive anticoagulant
White blood cell count that exceeds linearity limits
Lipemic plasma
Scratched or dirty hemoglobin measuring cell

A

Excessive anticoagulant

Excessive anticoagulant does not affect hemoglobin readings.

Anything that causes an increase in absorbance will cause a hemoglobin that is read spectrophotometrically to be falsely high.

371
Q

Blood drop size in the manual wedge technique:

1 to 2 mm in diameter
2 to 3 mm in diameter
4 to 5 mm in diameter
5 to 6 mm in diameter

A

2 to 3 mm in diameter

TOO LARGE A DROP: Long or thick film
TOO SMALL A DROP: Short or thin film

372
Q

In manual wedge technique, the PUSHER SLIDE is held securely in the dominant hand at about:

15 to 20 degree angle
30 to 45 degree angle
45 to 50 degree angle
80 to 90 degree angle

A

30 to 45 degree angle

373
Q

In the preparing wedge smear from blood samples of polycythemic patients, the angle between the two slides should be:

25
30
35
45

A

25

When the hematocrit is higher than normal (i.e., 60%), as is found in patients with polycythemia or in newborns, the angle should be lowered (i.e., 25 degrees) so the film is not too short and thick.

374
Q

When a blood film is viewed through the microscope, the RBCs appear redder than normal, the neutrophils are barely visible, and the eosinophils are bright orange. What is the most likely cause?

Slide was overstained
Stain was too alkaline
Buffer was too acidic
Slide was not rinsed adequately

A

Buffer was too acidic

EXCESSIVELY PINK STAIN
Insufficient staining
Prolonged washing time
Mounting the coverslips before they are dry
Too high acidity of the stain or buffer

EXCESSIVELY BLUE STAIN
Thick films
Prolonged staining time
Inadequate washing
Too high an alkalinity of stain or

375
Q

Detects lymphocytic cells and certain abnormal erythrocytic cells by staining of cytoplasmic glycogen:

Periodic acid Schiff (PAS)
Peroxidase
Sudan black B (SBB)
Toluidine blue

A

Periodic acid Schiff (PAS)

376
Q

Counting area for manual RBC count:

0.2 mm2
1 mm2
4 mm2

A

0.2 mm2

377
Q

Counting area for manual WBC count:

0.2 mm2
1 mm2
4 mm2

A

4 mm2

378
Q

Counting area for manual platelet count:

0.2 mm2
1 mm2
4 mm2

A

1 mm2

379
Q

To increase accuracy of the WBC differential, it is advisable to count at least ___ cells when the WBC count is higher than 40 x 10 9th/L.

Count 50 cells
Count 100 cells
Count 200 cells
Count 300 or 400 cells

A

Count 200 cells

380
Q

In WBC differential, if the WBC count is 100 x 10 9th/L or greater, it would be more precise and accurate to count ____ cells.

Count 50 cells
Count 100 cells
Count 200 cells
Count 300 or 400 cells

A

Count 300 or 400 cells

381
Q

Effect of excess anticoagulant to hematocrit:

Increase
Decrease
No effect
Cannot be determined

A

Decrease

382
Q

Which of the following will stain the GRANULOCYTES only?

Naphthol AS-D chloroacetate
Alpha-naphthyl acetate esterase
Alpha-naphthyl butyrate esterase
All of these

A

Naphthol AS-D chloroacetate

383
Q

Dehydration:

Decreased hematocrit
Increased hematocrit
Variable hematocrit
Hematocrit cannot be determined

A

Increased hematocrit

The fluid loss associated with dehydration causes a decrease in plasma volume and falsely INCREASES the hematocrit reading.

384
Q

Myeloperoxidase (MPO) and Sudan Black B (SBB) staining results:

Opposite
Parallel
Undetermined
Variable

A

Parallel

385
Q

Which mutation is shared by a large percentage of patients with polycythemia vera, essential thrombocythemia, and primary myelofibrosis?

BCR/ABL
JAK2 V617F
PDGFR
RUNX1

A

JAK2 V617F

The JAK2 V617F mutation is present in numerous cases of myeloproliferative neoplasms, including polycythemia vera, essential thrombocythemia, and primary myelofibrosis.

BCR/ABL mutations are seen in CML, and PDGFR mutations are seen in neoplasms with eosinophilia. RUNX mutations may be seen in some cases of acute leukemias.

386
Q

A patient has a CBC and peripheral smear with an elevated WBC count and left shift, suggestive of a diagnosis of CML. Which of the following tests would be the most helpful in CONFIRMING the suspected diagnosis?

Cytochemical staining for myeloperoxidase and LAP
Karyotyping for the Philadelphia chromosome
Flow cytometry for myeloid cell markers
Lymph node biopsies for metastasis

A

Karyotyping for the Philadelphia chromosome

Although leukocyte alkaline phosphatase (LAP) scores tend to be decreased in chronic myelogenous leukemia (CML) and myeloid cells are present, karyotyping for the presence of the Philadelphia chromosome (9;22 translocation) is required for the confirmation of a diagnosis of CML.

387
Q

A 75-year-old male patient visits his physician for an annual checkup. His CBC showed an elevated WBC count with numerous small lymphocytes and smudge cells, and a subsequent bone marrow biopsy and aspirate showed hypercellularity with increased lymphoid cells. What is a presumptive diagnosis based on this information? *

Acute lymphoblastic leukemia
Chronic lymphocytic leukemia
Hairy cell leukemia
Therapy-related acute myelogenous leukemia

A

Chronic lymphocytic leukemia

This smear and bone marrow picture is typical of chronic lymphocytic leukemia (CLL) with numerous mature small lymphocytes.

388
Q

Which of the following appearances describes the types of cells seen in Sezary syndrome?

Plasma cells containing immunoglobulin deposits
Large circulating micromegakaryocytes
Lymphocytes with convoluted, cerebriform nuclei
Prolymphocytes with prominent azurophilic granules

A

Lymphocytes with convoluted, cerebriform nuclei

Sezary cells are lymphoma cells characterized by cerebriform nuclei and irregular nuclear outlines.

389
Q

A patient’s differential count shows an elevated eosinophil count. This is consistent with which of the following?

Aplastic anemia
Bacterial infection
Parasitic infection
Viral infection

A

Parasitic infection

Elevated eosinophil counts are often seen in parasitic infections, particularly those caused by helminths.

390
Q

A patient has a splenomegaly, and his CBC shows a left shift; bizarre RBCs, including dacryocytes; and notable platelet abnormalities. Which of the following would be the most helpful in determining the patient’s diagnosis?

Bone marrow biopsy
LAP staining
Karyotyping for the Philadelphia chromosome
Spleen biopsy

A

Bone marrow biopsy

A bone marrow biopsy would help in confirming a diagnosis of primary myelofibrosis, which is a possible diagnosis suggested by the dacryocytes, left shift, and abnormal platelets. If the patient had primary myelofibrosis, the bone marrow would likely show areas of fibrosis, in addition to increases in megakaryocytes and abnormal platelets.

391
Q

Lipemia can cause turbidity in the cyanmethemoglobin method and a falsely high hemoglobin result. It can be corrected by:

-Reagent-sample solution can be centrifuged and the supernatant measured
-Adding 0.01 mL of the patient’s plasma to 5 mL of the cyanmethemoglobin reagent and using this solution as the reagent blank
-Making a 1:2 dilution with distilled water (1 part diluted sample plus 1 part water) and multiplying the results from the standard curve by 2.
-Cannot be corrected

A

Adding 0.01 mL of the patient’s plasma to 5 mL of the cyanmethemoglobin reagent and using this solution as the reagent blank

392
Q

If FEWER THAN 50 PLATELETS are counted on each side, the procedure should be repeated by diluting the blood to:

1:10
1:20
1:100
1:200

A

1:20

393
Q

If MORE THAN 500 PLATELETS are counted on each side a ____ dilution should be made.

1:10
1:20
1:100
1:200

A

1:200

394
Q

WBC COUNT that can cause turbidity in the cyanmethemoglobin method resulting to falsely high hemoglobin value:

Greater than 5 x 10 9th/L
Greater than 10 x 10 9th/L
Greater than 15 x 10 9th/L
Greater than 20 x 10 9th/L

A

Greater than 20 x 10 9th/L

A high WBC count (greater than 20 x 10 9th/L) or a high platelet count (greater than 700 x 10 9th/L) can cause turbidity and a falsely high result. IN THIS CASE, THE REAGENT-SAMPLE SOLUTION CAN BE CENTRIFUGED AND THE SUPERNATANT MEASURED.

395
Q

PLATELET COUNT that can cause turbidity in the cyanmethemoglobin method resulting to falsely high hemoglobin value:

Greater than 150 x 10 9th/L
Greater than 200 x 10 9th/L
Greater than 500 x 10 9th/L
Greater than 700 x 10 9th/L

A

Greater than 700 x 10 9th/L

A high WBC count (greater than 20 x 10 9th/L) or a high platelet count (greater than 700 x 10 9th/L) can cause turbidity and a falsely high result. IN THIS CASE, THE REAGENT-SAMPLE SOLUTION CAN BE CENTRIFUGED AND THE SUPERNATANT MEASURED.

396
Q

When the correct area of a specimen from a patient with a normal RBC count is viewed, there are generally about ____ RBCs per 100x oil immersion field.

10 to 15 RBCs per OIF
20 to 25 RBCs per OIF
100 to 150 RBCs per OIF
200 to 250 RBCs per OIF

A

200 to 250 RBCs per OIF

397
Q

The IMPEDANCE PRINCIPLE OF CELL COUNTING is based on the detection and measurement of CHANGES IN ELECTRICAL RESISTANCE produced by cells as they traverse a small aperture. Electrical resistance between the ___ electrodes, or impedance in the current, occurs as the cells pass through the sensing aperture, causing voltage pulses that are measurable.

Two (2) electrodes
Three (3) electrodes
Four (4) electrodes
Ten (10) electrodes

A

Two (2) electrodes

398
Q

Forward, low angle light scatter:

0 degree angle
2 to 3 degree angle
5 to 15 degree angle
90 degree angle

A

2 to 3 degree angle

Forward-angle: 0 degree - cell volume
Forward low-angle: 2 to 3 degrees - cell volume
Forward high-angle: 5 to 15 degrees - internal complexity
Orthogonal, side: 90 degrees - internal complexity

399
Q

Forward, high angle light scatter:

0 degree angle
2 to 3 degree angle
5 to 15 degree angle
90 degree angle

A

5 to 15 degree angle

Forward-angle: 0 degree - cell volume
Forward low-angle: 2 to 3 degrees - cell volume
Forward high-angle: 5 to 15 degrees - internal complexity
Orthogonal, side: 90 degrees - internal complexity

400
Q

Erythrocytes with NORMAL RDW:

-Homogeneous in character and exhibit very little anisocytosis on a peripheral blood smear
-Heterogeneous and exhibit a high degree of anisocytosis

A

Homogeneous in character and exhibit very little anisocytosis on a peripheral blood smear

401
Q

Erythrocytes with INCREASED RDW:

-Homogeneous in character and exhibit very little anisocytosis on a peripheral blood smear
-Heterogeneous and exhibit a high degree of anisocytosis

A

Heterogeneous and exhibit a high degree of anisocytosis

402
Q

THE ESR IS ____ PROPORTIONAL TO THE RED BLOOD CELL MASS and _____proportional to plasma viscosity.

Direct, direct
Direct, inverse
Inverse, direct
Inverse, inverse

A

Direct, inverse

403
Q

In automated cell counters, these parameters are directly measured:

WBC and RBC
WBC, RBC, and hemoglobin
WBC and hemoglobin
RBC only

A

WBC, RBC, and hemoglobin

404
Q

All of the following are causes of spurious increase in WBCs, except:

Cryoglobulin
Cryofibrinogen
Monoclonal proteins
Smudge cells

A

Smudge cells

SPURIOUS INCREASE IN WBC
Cryoglobulin, cryofibrinogen
Heparin, monoclonal proteins
Nucleated RBCs, unlysed RBCs
Platelet clumping

SPURIOUS DECREASE IN WBC
Clotting, smudge cells
Uremia plus immunosuppressants

405
Q

All of the following are causes of spurious increase in RBCs, except:

Autoagglutination
Cryoglobulin, cryofibrinogen
Giant platelets
High WBCs

A

Autoagglutination

SPURIOUS INCREASE IN RBC
Cryoglobulin, cryofibrinogen
Giant platelets
High WBC (>50,000/mL)

SPURIOUS DECREASE IN RBC
Autoagglutination, clotting
Hemolysis (in vitro)
Microcytic red cells

406
Q

Based on the detection and measurement of changes in electrical resistance produced by cells as they transverse a small aperture in a conducting solution:

Impedance principle of cell counting
Optical principle of cell counting

A

Impedance principle of cell counting

407
Q

Based on the degree of scatter and the amount of light reaching the sensor depend on the volume of the cell:

Impedance principle of cell counting
Optical principle of cell counting

A

Optical principle of cell counting

408
Q

Cold agglutinins:

Decreased RBCs, increased MCV and MCHC, grainy appearance
Increased RBCs, decreased MCV and MCHC, grainy appearance

A

Decreased RBCs, increased MCV and MCHC, grainy appearance

RATIONALE: Agglutination of RBCs
CORRECTIVE ACTION: Warm specimen to 37° C and rerun

409
Q

Lipemia and icterus:

Decreased hemoglobin and MCH
Increased hemoglobin and MCH

A

Increased hemoglobin and MCH

RATIONALE: Turbidity affects spectrophotometric reading for hemoglobin
CORRECTIVE ACTION: Plasma replacement

410
Q

Nucleated RBCs, megakaryocyte fragments, or micromegakaryoblasts:

Decreased WBCs in newer instruments
Increased WBCs in older instruments

A

Increased WBCs in older instruments

RATIONALE: Nucleated RBCs or micromegakaryoblasts counted as WBCs
CORRECTIVE ACTION: Newer instruments eliminate this error and count nucleated RBCs and correct the WBC count; count micromegakaryoblasts per 100 WBCs and correct

411
Q

Platelet clumps:

Decreased platelets, increased WBCs
Increased platelets, decreased WBCs

A

Decreased platelets, increased WBCs

RATIONALE: Large clumps counted as WBCs and not platelets
CORRECTIVE ACTION: Redraw specimen in sodium citrate, multiply result by 1.1

412
Q

Leukemia, especially with chemotherapy:

Decreased WBCs, increased platelets
Increased WBCs, decreased platelets

A

Decreased WBCs, increased platelets

RATIONALE Fragile WBCs, fragments counted as platelets
CORRECTIVE ACTION Review film, perform phase platelet count or CD61 count

413
Q

WBC > 100,000/µL:

Decreased hemoglobin, decreased RBCs, incorrect hematocrit
Increased hemoglobin, increased RBCs, incorrect hematocrit

A

Increased hemoglobin, increased RBCs, incorrect hematocrit

RATIONALE: Turbidity affects spectrophotometric reading for hemoglobin, WBCs counted with RBC count
CORRECTIVE ACTION: Manual HCT; perform manual HGB (spin/read supernatant), correct RBC count, recalculate indices; if above linearity, dilute for correct WBC count

414
Q

All of the following are causes of spurious decrease in MCHC, except: *

Autoagglutination
High WBCs
Spuriously low hemoglobin
Spuriously high hematocrit

A

Autoagglutination

SPURIOUS INCREASE IN MCHC
Autoagglutination, clotting
Hemolysis (in vitro, in vivo)
Spuriously HIGH Hb
Spuriously LOW Hct

SPURIOUS DECREASE IN MCHC
High WBC (>50,000/μL)
Spuriously LOW Hb,
Spuriously HIGH Hct

415
Q

Shift to the left in RBC histogram means:

Population of smaller cells (microcytosis)
Population of larger cells (macrocytosis)

A

Population of smaller cells (microcytosis)

416
Q

A stained blood film is held up to the light and observed to be bluer than normal. What microscopic abnormality might be expected on this film?

Rouleaux
Spherocytosis
Reactive lymphocytosis
Toxic granulation

A

Rouleaux

MACROSCOPIC EXAMINATION OF BLOOD FILM
1. BLUER
Increased blood proteins, as in plasma cell myeloma, and that ROULEAUX may be seen on the film

  1. GRAINY
    RBC agglutination, as in cold hemagglutinin diseases
  2. HOLES Increased lipid levels
  3. BLUE SPECKS OUT AT THE FEATHER EDGE
    Markedly increased WBC counts and platelet counts
417
Q

Holes all over the blood film indicates:

RBC agglutination
Markedly increased WBC and platelet counts
Increased lipid levels
Increased blood proteins as in multiple myeloma

A

Increased lipid levels

MACROSCOPIC EXAMINATION OF BLOOD FILM
1. BLUER
Increased blood proteins, as in plasma cell myeloma, and that ROULEAUX may be seen on the film

  1. GRAINY
    RBC agglutination, as in cold hemagglutinin diseases
  2. HOLES Increased lipid levels
  3. BLUE SPECKS OUT AT THE FEATHER EDGE
    Markedly increased WBC counts and platelet counts
418
Q

Which of the following tests is the most effective means of assessing red blood cell generation in response to anemia?

RDW
Reticulocyte count
Platelet count
CB

A

Reticulocyte count

419
Q

Which of the following conditions is not usually associated with marked reticulocytosis?

Four days after a major hemorrhage
Drug-induced autoimmune hemolytic anemia
Sickle cell anemia
Pernicious anemia

A

Pernicious anemia

The reticulocyte count is not usually elevated in pernicious anemia even though increased marrow erythropoiesis occurs. The defective cellular maturation that occurs in megaloblastic anemias results in the death of many red cells in the bone marrow (ineffective erythropoiesis).

420
Q

If 60 reticulocytes are counted in 1000 red blood cells, what is the reticulocyte count?

0.06%
0.6%
6.0%
60.0%

A

6.0%

421
Q

To improve accuracy of the RETICULOCYTE COUNT, have another laboratorian count the other film; counts should agree within:

20%
30%
40%
50%

A

20%

422
Q

Which of the following is NOT associated with causing a falsely low ESR?

Column used is slanted
EDTA tube is clotted
EDTA tube is one-third full
EDTA specimen is 24 hours old

A

Column used is slanted

423
Q

The reagent used in the traditional sickle cell screening test is:

Sodium chloride
Sodium citrate
Sodium metabisulfite
Sodium-potassium oxalate

A

Sodium metabisulfite

424
Q

Hemoglobin SOLUBILITY TEST is a screening test for:

Hemoglobin A2
Hemoglobin F
Hemoglobin S
Unstable hemoglobins

A

Hemoglobin S

425
Q

Reagent for the solubility test:

Hydrochloric acid
Ferric cyanide and detergent
2% sodium metabisulfite
Sodium hydrosulfite (dithionite) and a detergent-based lysing agent (saponin)

A

Sodium hydrosulfite (dithionite) and a detergent-based lysing agent (saponin)

426
Q

Positive result in the solubility test (sodium dithionite test for hemoglobin S):

Normal looking RBCs or slightly crenated RBCs
Sickle cells or ‘holly leaf’ form of RBCs
Clear solution, black lines of the reader scale are visible through the solution
Turbid solution, reader scale is not visible through the test solution

A

Turbid solution, reader scale is not visible through the test solution

If the solubility test is positive, hemoglobin electrophoresis should be performed on the specimen.

427
Q

Paroxysmal nocturnal hemoglobinuria is characterized by flow cytometry results that are:

Negative for CD55 and CD59
Positive for CD55 and CD59
Negative for CD4 and CD8
Positive for all normal CD markers

A

Negative for CD55 and CD59

428
Q

If the sugar water test is positive, _____ procedure should be performed before a diagnosis of PNH is made.

Autohemolysis test
Hemoglobin electrophoresis
Osmotic fragility test
Sucrose hemolysis test

A

Sucrose hemolysis test

429
Q

Anticoagulant for the sugar water and sucrose hemolysis test:

EDTA
Citrate
Heparin
Oxalate

A

Citrate

430
Q

What are the INITIAL laboratory tests that are performed for the diagnosis of anemia?

CBC, iron studies, and reticulocyte count
CBC, reticulocyte count, and peripheral blood film examination
Reticulocyte count and serum iron, vitamin B12 and folate assays
Bone marrow study, iron studies, and peripheral blood film examination

A

CBC, reticulocyte count, and peripheral blood film examination

431
Q

Dilution for RBC count in AUTOMATED cell counters:

1:100
1:200
1: 50,000
1: 500

A

1: 50,000

432
Q

Dilution for WBC count in AUTOMATED cell counters:

1:100
1:200
1: 50,000
1: 500

A

1: 500

433
Q

A decreased osmotic fragility test would be associated which of the following conditions?

Sickle cell anemia
Hereditary spherocytosis
HDN
Acquired hemolytic anemia

A

Sickle cell anemia

434
Q

A Miller disk is an ocular device used to facilitate counting of:

Platelets
Reticulocytes
Sickle cells
Nucleated RBCs

A

Reticulocytes

435
Q

The Coulter principle for counting of cells is based upon the fact that:

Isotonic solutions conduct electricity better than cells do
Conductivity varies proportionally to the number of cells
Cells conduct electricity better than saline does
Isotonic solutions cannot conduct electricity

A

Isotonic solutions conduct electricity better than cells do

436
Q

Side angle scatter in a laser-based cell counting system is used to measure:

Cell size
Cytoplasmic granularity
Cell number
Immunologic (antigenic) identification

A

Cytoplasmic granularity

437
Q

Falsely elevated automated platelet counts may result from:

Platelet satellitism
Platelet agglutinins
Exceptionally large platelets
Erythrocyte inclusion bodies

A

Erythrocyte inclusion bodies

438
Q

The mean platelet volume (MPV):

Analogous to the MCHC
Direct measure of the platelet count
Measurement of the average volume of platelets
Comparison of the patient’s value to the normal value

A

Measurement of the average volume of platelets

439
Q

The RDW and MCV are both quantitative descriptors of erythrocyte size. If both are increased, the most probable erythrocytic abnormality would be:

Iron deficiency anemia
Acquired aplastic anemia
Megaloblastic anemia
Hemoglobinopathy

A

Megaloblastic anemia

440
Q

In an erythrocyte histogram, the erythrocytes that are larger than normal will be to the _____ of the normal distribution curve.

Right
Left
In the middle

A

Right

441
Q

If the RBC distribution on a histogram demonstrates a homogeneous pattern and a small SD, the peripheral blood smear would probably exhibit:

Extreme anisocytosis
Very little anisocytosis
A single population of spherocytes
A single population of macrocytes

A

Very little anisocytosis

442
Q

Which of the following characteristics are common to hereditary spherocytosis, hereditary elliptocytosis, hereditary stomatocytosis, and paroxysmal nocturnal hemoglobinuria?

Autosomal dominant inheritance
Red cell membrane defects
Positive direct antiglobulin test
Measured platelet count

A

Red cell membrane defects

443
Q

An enzyme deficiency associated with a moderate to severe hemolytic anemia after the patient is exposed to certain drugs and characterized by red cell inclusions formed by denatured hemoglobin is:

Lactate dehydrogenase deficiency
G-6-PD deficiency
Pyruvate kinase deficiency
Hexokinase deficiency

A

G-6-PD deficiency

444
Q

The presence of excessive rouleaux formation on a blood smear is often accompanied by an increased:

Reticulocyte count
Sedimentation rate
Hematocrit
Erythrocyte count

A

Sedimentation rate

445
Q

MPV values should be based on specimens that are between ____ and ___ hours old.

1 and 4 hours old
4 and 6 hours old
6 and 8 hours old
16 and 18 hours old

A

1 and 4 hours old

MPV values should be based on specimens that are between 1 and 4 hours old.

446
Q

Complications include orthostatic hypotension, syncope and shock *

Vascular complications
Cardiovascular complications
Neurological complications
Dermatological comlications

A

Cardiovascular complications

Cardiovascular complications include orthostatic hypotension, syncope, shock, and cardiac arrest.

447
Q

Complications include diaphoresis, seizure and pain: *

Vascular complications
Cardiovascular complications
Neurological complications
Dermatological comlications

A

Neurological complications

Neurological Complications Post-phlebotomy patients can exhibit some neurological complications. These include diaphoresis, seizure, pain, and nerve damage. A physician should be consulted immediately.

448
Q

Sister chromatids move to the equatorial plate. *

Prophase
Metaphase
Anaphase
Telophase

A

Metaphase

Characteristics of the Four Mitotic Periods

PROPHASE
The chromatin becomes tightly coiled.
Nucleolus and nuclear envelope disintegrate.
Centrioles move to opposite poles of the cell.

METAPHASE
Sister chromatids move to the equatorial plate.

ANAPHASE
Sister chromatids separate and move to opposite poles.

TELOPHASE
Chromosomes arrive at opposite poles.
Nucleolus and nuclear membrane reappear.
The chromatin pattern reappears.

449
Q

Megakaryocytes develop into platelets in approximately __ days. *

3 days
5 days
9 days
12 days

A

5 days

Megakaryocytes develop into platelets in approximately 5 days.

450
Q

This cytokine promotes the growth of early hematopoietic cell lines: *

Interleukin 1
Interleukin 2
Interleukin 3
Interleukin 6

A

Interleukin 3

Promotes the growth of early hematopoietic cell lines (e.g., proliferation of CFU-GEMM, CFU-M, CFU-Meg, CFU-Eo, and CFU-Bs colonies from
bone marrow).

IL-3 acts with M-CSF to stimulate proliferation of monocytes and macrophages. It also stimulates granulocyte, monocyte, eosinophil,
and mast cell production

451
Q

Hemoglobin appears for the first time: *

Rubriblast (pronormoblast)
Prorubricyte (basophilic normoblast)
Rubricyte (polychromatophilic normoblast)
Metarubricyte (orthochromic normoblast)

A

Rubricyte (polychromatophilic normoblast)

Hemoglobin appears for the first time in the third maturational stage, the rubricyte or polychromatic normoblast.

452
Q

This pathway prevents denaturation of globin of the hemoglobin molecule by oxidation: *

Embden-Meyerhof pathway
Hexose-monophosphate shunt
Methemoglobin reductase pathway
Luebering-Rapoport pathway

A

Hexose-monophosphate shunt

Embden-Meyerhof Pathway
Maintains cellular energy by generating ATP

Oxidative pathway or hexose-monophosphate shunt
Prevents denaturation of globin of the hemoglobin molecule by oxidation

Methemoglobin reductase pathway
Prevents oxidation of heme iron

Luebering-Rapaport pathway
Regulates oxygen affinity of hemoglobin

453
Q

RBCs inclusions, 0.2 to 2.0 mm in size, that can be seen with a stain such as crystal violet or brilliant cresyl blue; represent precipitated, denatured hemoglobin and are clinically associated with congenital hemolytic anemia, G6PD deficiency, hemolytic anemias secondary to drugs such as phenacetin, and some hemoglobinopathies. *

Hemoglobin C crystals
Heinz bodies
Howell-Jolly bodies
Pappenheimer bodies

A

Heinz bodies

Heinz bodies are inclusions, 0.2 to 2.0 mm in size, that can be seen with a stain such as crystal violet or brilliant cresyl blue. They represent precipitated, denatured hemoglobin and are clinically associated with congenital hemolytic anemia, G6PD deficiency, hemolytic anemias secondary to drugs such as phenacetin, and some hemoglobinopathies.

454
Q

RBC inclusions that are aggregates of mitochondria, ribosomes, and iron particles. Clinically, they are associated with iron-loading anemias, hyposplenism, and hemolytic anemias. *

Basophilic stippling
Heinz bodies
Howell-Jolly bodies
Pappenheimer bodies

A

Pappenheimer bodies

455
Q

RBC inclusions representing granules composed of ribosomes and RNA that are precipitated during the process of staining of a blood smear; associated clinically with disturbed erythropoiesis (defective or accelerated heme synthesis),lead poisoning, and severe anemias. *

Basophilic stippling
Heinz bodies
Howell-Jolly bodies
Pappenheimer bodies

A

Basophilic stippling

456
Q

Nuclear remnants predominantly composed of DNA; believed to develop in periods of accelerated or abnormal erythropoiesis, because the spleen cannot keep upwith pitting these remnants from the cell. Its presence is associated with hemolytic anemias, pernicious anemia, and particularly post-splenectomy, physiologicalatrophy of the spleen. *

Basophilic stippling
Heinz bodies
Howell-Jolly bodies
Pappenheimer bodies

A

Howell-Jolly bodies

Howell-Jolly bodies are believed to develop in periods of accelerated or abnormal erythropoiesis, because the spleen cannot keep up with pitting these remnants from the cell. The presence of Howell-Jolly bodies is associated with hemolytic anemias, pernicious anemia, and particularly post-splenectomy, physiological atrophy of the spleen.

457
Q

Anemias with low MCV and MCHC; microcytic, hypochromic RBCs, EXCEPT: *

Iron deficiency anemia
Thalassemia
Sideroblastic anemia
Excessive alcohol ingestion

A

Excessive alcohol ingestion

Low MCV, MCHC
Microcytic, hypochromic
Typical of maturation defects
Iron deficiency anemia (some)
Thalassemia
Sideroblastic anemia

Normal MCV, MCHC
Normocytic, normochromic
Typical of hypoproliferation
Bone marrow disorder
Iron deficiency anemia (some)
Anemia of chronic disorders
Autoimmune disease

High MCV
Macrocytic
Typical of maturation defect
Vitamin B12 deficiency
Folate deficiency
Excessive alcohol ingestion
Hypothyroidism

458
Q

Severe increase in abnormal erythrocytes in each microscopic field; an equivalent descriptive term is MANY. *

0
1+
2+
3+
4+

A

3+

Grading of Erythrocyte Morphology
0 Normal appearance or slight variation in erythrocytes.
1+ Only a small population of erythrocytes displays a particular abnormality; the terms slightly increased or few would be comparable.
2+ More than occasional numbers of abnormal erythrocytes can be seen in a microscopic field; an equivalent descriptive term is moderately increased.
3+ Severe increase in abnormal erythrocytes in each microscopic field; an equivalent descriptive term is many.
4+ The most severe state of erythrocytic abnormality, with the abnormality prevalent throughout each microscopic field; comparable terms are marked or marked increase.

459
Q

Inherited hemolytic anemia due to structural membrane defect: *

Thalassemia
Sickle cell anemia
Pyruvate kinase deficiency
Hereditary spherocytosis

A

Hereditary spherocytosis

Examples of Inherited Hemolytic Anemias

STRUCTURAL MEMBRANE DEFECTS
Acanthocytosis
Hereditary spherocytosis
Hereditary elliptocytosis
Hereditary stomatocytosis
Hereditary xerocytosis
Rh null disease

ERYTHROCYTIC ENZYME DEFECTS
G6PD deficiency
Glutathione reductase
Hexokinase
Pyruvate kinase

DEFECTS OF THE HEMOGLOBIN MOLECULE
Hb C disorder
Hb S-C disorder
Hb S-S disorder (sickle cell anemia)
Thalassemia

460
Q

Hemoglobinopathies associated with ABNORMAL MOLECULAR STRUCTURE: *

Alpha thalassemia
Alpha and beta thalassemia
Sickle cell anemia and beta thalassemia
Sickle cell anemia, sickle cell trait and Hb C disease

A

Sickle cell anemia, sickle cell trait and Hb C disease

Examples of Selected Hemoglobinopathies
ABNORMAL MOLECULAR STRUCTURE
Hb SS (sickle cell anemia)
Hb SA (sickle cell trait)
Hb C disease or trait

RATE OF SYNTHESIS
Beta-Thalassemia
Alpha-Thalassemia

COMBINATION OF TWO MOLECULAR ALTERATIONS OR A MOLECULAR ABNORMALITY AND SYNTHESIS DEFECT
Hb S-Hb C
Hb S-b-thalassemia

461
Q

Elongated and curved nucleus; very clumped chromatin: *

Myelocyte
Metamyelocyte
Band
Segmenter neutrophil

A

Band

462
Q

Indented nucleus, clumped chromatin: *

Promyelocyte
Myelocyte
Metamyelocyte
Band

A

Metamyelocyte

463
Q

Mast cells have an appearance similar to that of the blood: *

Monocyte
Neutrophil
Eosinophil
Basophil

A

Basophil

Mast cells (tissue basophils) are not observed in the blood of healthy persons. These cells have an appearance similar to that of the blood basophil. Mast cells have a round or oval nucleus. The granules of the mast cell do not overlie the nucleus as they do in basophils.

464
Q

Once the metamyelocyte stage has been reached, cells have undergone ____ cell divisions and the proliferative phase comes to an end. *

1 or 2 cell divisions
2 or 3 cell divisions
4 or 5 cell divisions
6 or 7 cell divisions

A

4 or 5 cell divisions

Once the metamyelocyte stage has been reached, cells have undergone four or five cell divisions and the proliferative phase comes to an end.

465
Q

Nuclear chromatin is coarse and clumped; dark blue (basophilic) cytoplasm around the periphery or in a radial pattern and few cytoplasmic vacuoles: *

Type I Downey cells
Type II Downey cells
Type III Downey cells

A

Type II Downey cells

Descriptive Features of the Classic Downey Classification of Lymphocytes Seen in Infectious Mononucleosis

Type I
Nucleus May be irregularly shaped
Cytoplasm Usually many cytoplasmic vacuoles, dark blue (basophilic)

Type II
Nucleus Chromatin is coarse and clumped
Cytoplasm Increased amount, dark blue (basophilic) around the periphery or in a radial pattern, a few cytoplasmic vacuoles

Type IIIa
Nucleus Nucleoli usually visible, enlarged in size
Cytoplasm Dark blue (basophilic)

466
Q

Myeloid cells demonstrate maturation beyond the blast and promyelocyte stage: *

M0 myeloid
M1 myeloid
M2 myeloid
M3 myeloid

A

M2 myeloid

M2 myeloidMyeloid cells demonstrate maturation beyond the blast and promyelocyte stage

467
Q

Abnormal proliferation of both erythroid and granulocytic precursors; may include abnormal megakaryocytic and monocytic proliferations: *

M3
M4
M6
M7

A

M6

M6 erythroleukemiaAlso known as Di Guglielmo syndrome; abnormal proliferation of both erythroid and granulocytic precursors; may include abnormal megakaryocytic and monocytic proliferations

468
Q

Small cells predominant; nuclear shape is regular with an occasional cleft; chromatin pattern is homogeneous and nucleoli are rarely visible; cytoplasm is moderately basophilic: *

L1
L2
L3

A

L1

L1 homogeneous
One population of cells within the case; small cells predominant; nuclear shape is regular with an occasional cleft; chromatin pattern is homogeneous and nucleoli are rarely visible; cytoplasm is moderately basophilic

L2 heterogeneous
Large cells with an irregular nuclear shape; clefts in the nucleus are common; one or more large nucleoli are visible; cytoplasm varies in color

L3 Burkitt lymphoma type
Cells are large and homogeneous in size; nuclear shape is round or oval; one to three prominent nucleoli; cytoplasm is deeply basophilic with vacuoles often prominent

469
Q

Cells are large and homogeneous in size; nuclear shape is round or oval; one to three prominent nucleoli; cytoplasm is deeply basophilic with vacuoles often prominent: *

L1
L2
L3

A

L3

L1 homogeneous
One population of cells within the case; small cells predominant; nuclear shape is regular with an occasional cleft; chromatin pattern is homogeneous and nucleoli are rarely visible; cytoplasm is moderately basophilic

L2 heterogeneous
Large cells with an irregular nuclear shape; clefts in the nucleus are common; one or more large nucleoli are visible; cytoplasm varies in color

L3 Burkitt lymphoma type
Cells are large and homogeneous in size; nuclear shape is round or oval; one to three prominent nucleoli; cytoplasm is deeply basophilic with vacuoles often prominent

470
Q

Leukemic reticuloendotheliosis: *

Prolymphocytoc leukemia
Plasma cell leukemia
Hairy cell leukemia
Sezary syndrome

A

Hairy cell leukemia

471
Q

Solid tumor counterpart of acute lymphoblastic leukemia: *

Lymphoma, undifferentiated
Lymphoma, poorly differentiated leukemia
Lymphoma, well-differentiated leukemia
Chloroma granulocytic leukemia

A

Lymphoma, poorly differentiated leukemia

472
Q

Solid tumor counterpart of plasma cell leukemia: *

Reticulum cell sarcoma
Chloroma granulocytic leukemia
Myeloma
Lymphoma, undifferentiated

A

Myeloma

473
Q

A distinctive feature of the megakaryocyte: *

Multinucleated
Multilobular
Multinucleated and multilobular
None of these

A

Multilobular

A distinctive feature of the megakaryocyte is that it is multilobular, not multinucleated. The fully mature lobes of the megakaryocyte shed platelets from the cytoplasm on completion of maturation.

474
Q

A target INR range of ____ is recommended for most indications (e.g., treatment or prophylaxis of deep venous thrombosis [DVT], or prevention of further clotting in patients who have had a myocardial infarction). *

INR range of 1.0 to 2.0
INR range of 2.0 to 3.0
INR range of 2.5 to 3.5
INR range of 4.0 to 5.0

A

INR range of 2.0 to 3.0

A target INR range of 2.0 to 3.0 is recommended for most indications (e.g., treatment or prophylaxis of deep venous thrombosis [DVT], or prevention of further clotting in patients who have had a myocardial infarction).

An INR of 2.5 to 3.5 is recommended for patients with prosthetic heart valves.

When the INR is used to guide anticoagulant therapy, there are fewer bleeding events.

475
Q

The target INR for pulmonary embolism (PE) treatment is ___ for the duration of anticoagulation. *

1.0
1.5
2.5
3.0

A

3.0

The target INR for pulmonary embolism (PE) treatment is 3.0 for the duration of anticoagulation.

476
Q

The new types of thromboplastins for measuring the PT are mixtures of phospholipids and recombinantly derived _____ tissue factor. *

Rabbit
Pig
Horse
Human

A

Human

NEW THROMBOPLASTINS
The new types of thromboplastins for measuring the PT are mixtures of phospholipids and recombinantly derived human tissue factor.

Because the new thromboplastins are more sensitive (typical ISI, 1.0) than the traditional North American ones (ISIs, 1.8 to 3.0), the PTs for patients with inherited or acquired deficiencies of coagulation factors will be much more prolonged with use of the new reagents, although normal values may change minimally.

477
Q

Anticoagulant therapy: *

Falsely decreased D-dimer values
Falsely increased D-dimer values
No effect
Cannot be determined

A

Falsely decreased D-dimer values

Conditions That Can Generate Falsely Decreased or Falsely Increased D-Dimer Values
FALSELY DECREASED VALUES
1. Anticoagulant therapy
2. Smaller, older, nonprogressing thrombus

FALSELY INCREASED VALUES
1. Various disease states
2. Post-therapeutic clinical procedures

478
Q

Smallest platelets seen: *

Wiskott-Aldrich syndrome
May-Hegglin anomaly
Alport syndrome
Bernard-Soulier syndrome

A

Wiskott-Aldrich syndrome

479
Q

Glanzmann thrombasthenia and essential athrombia: *

Platelet adhesion defect
Primary platelet aggregation defect
Secondary platelet aggregation defect
Isolated platelet factor III deficicency

A

Primary platelet aggregation defect

Hereditary Platelet Function Defects
ADHESION DEFECTS
Bernard-Soulier syndrome
Impaired adhesion to collagen

AGGREGATION DEFECTS: PRIMARY
Glanzmann thrombasthenia
Essential athrombia

AGGREGATION DEFECTS: SECONDARY
Storage pool diseases
Aspirin-like defects
Release reaction defects

ISOLATED PLATELET FACTOR III DEFICIENCY
SEVERE COAGULATION FACTOR DEFICIENCIES

Afibrinogenemia
Factor VIII: C deficiency
Factor IX: C deficiency

480
Q

Acquired platelet function defects: *

Bernard-Soulier syndrome
Bernard-Soulier and Glanzmann thrombasthenia
Uremia, multiple myeloma
Uremia, multiple myeloma, vitamin B12 or folate deficiency

A

Uremia, multiple myeloma, vitamin B12 or folate deficiency

ACQUIRED PLATELET FUNCTION DEFECTS
1. Myeloproliferative syndromes
Essential thrombocythemia
Chronic myelogenous leukemia
Polycythemia vera
Paroxysmal nocturnal hemoglobinuria
Myelofibrosis
RAEB syndrome
Sideroblastic anemia
2. Paraprotein disorders
Multiple myeloma
Waldenström macroglobulinemia
Essential monoclonal gammopathy
3. Autoimmune diseases
Collagen vascular disease
Antiplatelet antibodies
Immune thrombocytopenias
4. Fibrinogen degradation products
Disseminated intravascular coagulation
Primary fibrinolytic syndromes
Liver disease
5. Anemia
Severe iron deficiency
Severe B12 or folate deficiency
6. Uremia
7. Drug induced
RAEB, refractory anemia with excess blasts

481
Q

Inherited platelet dysfunction: *

Bernard-Soulier syndrome
Bernard-Soulier syndrome, Glanzmann’s thrombasthenia
Uremia, multiple myeloma
Uremia, multiple myeloma, vitamin B12 or folate deficiency

A

Bernard-Soulier syndrome, Glanzmann’s thrombasthenia

INHERITED PLATELET DYSFUNCTION
1. Surface membrane defects
Bernard-Soulier syndrome
Glanzmann thrombasthenia
Platelet-type von Willebrand disease
2. Defects of granule storage
Alpha-granule deficiency
Gray platelet syndrome
3. Dense granules
Wiskott-Aldrich syndrome
Hermansky-Pudlak syndrome
Chédiak-Higashi syndrome
TAR baby syndrome

482
Q

Patients with _____ , the most severe form of von Willebrand disease, are likely to have a major episode of bleeding early in life because significantly decreased amounts of vWF and VIII:C are produced. *

Type IA
Type IIB
Type IIC, IID
Type III

A

Type III

Patients with type III, the most severe form of von Willebrand disease, are likely to have a major episode of bleeding early in life because significantly decreased amounts of vWF and VIII:C are produced.

483
Q

Conditions related to deficiencies of multiple coagulation factors: *

Hepatic disease
Hepatic disease and anticoagulant overdose
Anticoagulant overdose and vitamin K deficiency
Hepatic disease, anticoagulant overdose, DIC and vitamin K deficiency
Hepatic disease, anticoagulant overdose, DIC and vitamin

A

K deficiency

CONDITIONS RELATED TO DEFICIENCIES OF MULTIPLE COAGULATION FACTORS

Hepatic disease, anticoagulant overdose (e.g., heparin or warfarin), DIC, vitamin K deficiency

484
Q

Indented or twisted nucleus, lacy chromatin and gray-blue cytoplasm: *

Segmented neutrophil
Band neutrophil
Monocyte
Lymphocyte

A

Monocyte

485
Q

orward high-angle light scatter: *

0 degree angle
2 to 3 degree angle
5 to 15 degree angle
90 degree anglez

A

5 to 15 degree angle

Angles of Light Scatter
Various angles of light scatter can aid in cellular analysis.
1. Forward light scatter 0°. This is diffracted light, which relates to the volume of the cell.
2. Forward low-angle light scatter 2° to 3°. This characteristic can relate to size or volume.
3. Forward high angle 5° to 15°. This type of measurement allows for description of the refractive index of cellular components.
4. Orthogonal light scatter 90°. The result of this application of light scatter is the production of data based on reflection and refraction of internal components, which correlates with internal complexity

486
Q

RBC histogram to the LEFT: *

RBCs are larger than normal
RBCs are smaller than normal
Seen in megaloblastic anemia
Treated anemia

A

RBCs are smaller than normal

If the cells are smaller than normal, the curve will be more to the left, as in untreated iron deficiency anemia.

If the cells are larger than normal, the histogram curve will be more to the right, as in the megaloblastic anemias.

After appropriate treatment of the underlying cause of an anemia, the curve should move toward the normal range.

487
Q

Erythrocytes with an increased RDW; *

Homogenous in character, very little anisocytosis
Homogenous, high degree of anisocytosis
Heterogenous, very little anisocytosis
Heterogenous, high degree of anisocytosis

A

Heterogenous, high degree of anisocytosis

Erythrocytes with a normal RDW are homogeneous in character and exhibit very little anisocytosis on a peripheral blood smear.

Erythrocytes with an increased RDW are referred to as heterogeneous and exhibit a high degree of anisocytosis on a peripheral blood smear.

488
Q

MPV values should be based on specimens that are between ____ hours old. *

1 and 4 hours old
4 and 8 hours old
6 and 12 hours old
12 and 16 hours old

A

1 and 4 hours old

MPV is a measure of the average volume of platelets in a sample.

In EDTA-anti-coagulated blood, platelets undergo a change in shape. This alteration (swelling) causes the MPV to increase approximately 20% during the first hour. After this time, the size is stable for at least 12 hours; however, MPV values should be based on specimens that are between 1 and 4 hours old.

No single normal range exists. Patients with a lower platelet count normally have a higher MPV, and patients with a higher platelet count have a lower MPV.

Analysis of a nomogram demonstrates that an MPV between 9.0 and 9.8 fL is in the normal range, if the platelet count is normal. MPVs from 7.8 to 8.9 fL or from 9.9 to 12.0 fL may be in the normal range, depending on the platelet count.

DECREASED MPV:
1. Aplastic anemia
2. Megaloblastic anemia
3. Wiskott-Aldrich syndrome
4. After chemotherapy

INCREASED MPV:
1. Idiopathic thrombocytopenic purpura
2. After splenectomy
3. Sickle cell anemia

489
Q

Measure of the uniformity of platelet size in a blood specimen: *

Platelet adhesion
Platelet aggregation
Mean platelet volume (MPV)
Platelet distribution width (PDW)

A

Platelet distribution width (PDW)

The PDW is a measure of the uniformity of platelet size in a blood specimen. This parameter serves as a validity check and monitors false results. A normal PDW is less than 20%.

490
Q

Placement of fire extinguishers every ___ feet. *

75 feet
100 feet
125 feet
150 feet

A

75 feet

Placement of fire extinguishers every 75 feet. A distinct system for marking the locations of fire extinguishers enables quick access when they are needed. Fire extinguishers should be checked monthly and maintained annually.

Placement of manual fire alarm boxes near the exit doors. Travel distance should not exceed 200 feet.

491
Q

Surfaces in the specimen collection and processing area should be cleaned with: *

70% isopropyl alcohol.
1:10 bleach solution.
Soap and water.
Any of the above

A

1:10 bleach solution.

492
Q

Which of the following is a proper way to clean up a small blood spill that has dried on a countertop? *

Moisten it with a disinfectant and carefully absorb it with a paper towel.
Rub it with an alcohol pad, then wipe the area with a clean alcohol pad.
Scrape it into a biohazard bag and wash the surface with soap and water.
Use a disinfectant wipe and scrub it in ever-increasing concentric circles

A

Moisten it with a disinfectant and carefully absorb it with a paper towel.

493
Q

The following test orders for different patient shave been received at the same time. Which test would you collect first? *

Fasting glucose
STAT glucose in the ER
STAT hemoglobin in ICU
ASAP CBC in ICU

A

STAT glucose in the ER

ER stats typically have priority over other stats.

494
Q

A member of the clergy is with the patient when you arrive to collect a routine specimen. What should you do? *

Ask the patient’s nurse what you should do.
Come back after the clergy person has gone.
Fill out a form saying you were unable to collect the specimen.
Say “Excuse me, I need to collect a specimen from this patient.”

A

Come back after the clergy person has gone.

If a physician or a member of the clergy is with the patient, don’t interrupt. The patient’s time with these individuals is private and limited. If the draw is not stat, timed or other urgent priority, go draw another patient and check back after that. If that is the only patient, wait outside the room for a few minutes or go back to the lab and draw the specimen on the next sweep. (In any case, always make certain your actions follow facility policy.) If the request is stat, timed, or other urgent priority , excuse yourself, explain why you are there, and ask permission to proceed.

495
Q

The most common complication encountered in obtaining a blood specimen; it is caused by leakage of a small amount of blood in the tissue around the puncture site: *

Petechiae
Hematoma
Ecchymosis
Hemoconcentration

A

Ecchymosis

Ecchymosis (Bruise): Bruising is the most common complication encountered in obtaining a blood specimen. It is caused by leakage of a small amount of blood in the tissue around the puncture site.

Hematoma: A hematoma results when leakage of a large amount of blood around the puncture site causes the area to rapidly swell.

496
Q

A patient complains of extreme pain when you insert the needle during a venipuncture attempt. The pain does not subside, but the patient does not feel any numbness or burning sensation. You know the needle is in the vein because the blood is flowing into the tube. You have only two tubes to fill, and the first one is almost full. What should you do? *

-Ask the patient if he or she wants you to continue the draw
-Discontinue the draw and attempt collection at another site
-Distract the patient with small talk and continue the draw
-Tell the patient to hang in there as you have only one tube left

A

Discontinue the draw and attempt collection at another site

If marked or extreme pain occurs, or the patient asks you to remove the needle for any reason, the venipuncture should be terminated immediately, even if there are no other signs of nerve injury.

497
Q

Which type of patient is most likely to have an arteriovenous fistula or graft? *

Arthritic
Dialysis
Hospice
Wheelchair-bound

A

Dialysis

An arteriovenous (AV) shunt, fistula, or graft is the permanent surgical connection of an artery and vein by direct fusion (fistula), resulting in a bulging vein, or with a piece of vein or tubing (graft) that creates a loop under the skin. It is typically created to be used for dialysis, commonly joins the radial artery and cephalic vein above the wrist on the underside of the arm, and has a distinctive buzzing sensation called a “thrill” when palpated. A temporary shunt with tubing on the surface of the skin can also be created.

498
Q

Type of immersion oil with high viscosity and is used in brightfield and standard clinical microscopy. In hematology, this oil is routinely used. *

Type A
Type B
Type C

A

Type B

Three types of immersion oil, differing in viscosity, are employed in the clinical laboratory:

  1. Type A has very low viscosity and is used in fluorescence and darkfield studies.
  2. Type B has high viscosity and is used in brightfield and standard clinical microscopy. In hematology, this oil is routinely used.
  3. Type C has very high viscosity and is used with inclined microscopes with long-focus objective lenses and wide condenser gaps.
499
Q

The recommended cleaner for removing oil from objectives is: *

Benzene
Xylene
Water
70% alcohol or lens cleaner

A

70% alcohol or lens cleaner

Use solvent sparingly. The use of xylene is discouraged, because it contains a carcinogenic component (benzene). Xylene is also a poor cleaning agent, leaving an oily film on the lens. Lens cleaner or 70% isopropyl alcohol employed sparingly on a cotton applicator stick can be used to clean the objective lenses.

500
Q

Often the objects appear to have “haloes” surrounding them.

Brightfield microscope
Darkfield microscope
Phase-contrast microscope
Polarized light microscope

A

Phase-contrast microscope

This phase difference produces variation in light intensity from bright to dark, creating contrast in the image. Often the objects appear to have “haloes” surrounding them.

501
Q

True for PRECISION:

-Measure of agreement between an assay value and the theoretical “true value” of its analyte
-Magnitude of error separating the assay result from the true value
-Easy to define but difficult to establish and maintain
-Relatively easy to measure and maintain

A

Relatively easy to measure and maintain

Accuracy is easy to define but difficult to establish and maintain; precision is relatively easy to measure and maintain.
Precision is the expression of reproducibility or dispersion about the mean, often expressed as SD or CV%.

502
Q

Slope measures: *

Random error
Constant systematic error
Proportional systematic error
Constant and proportional systematic error

A

Proportional systematic error

Perfect correlation generates a slope of 1 and a y intercept of 0.

Slope measures proportional systematic error; the higher the analyte value, the greater the deviation from the line of identity. Proportional errors are caused by malfunctioning instrument components or a failure of some part of the testing process. The magnitude of the error increases with the concentration or activity of the analyte. An assay with proportional error may be invalid.

Intercept measures constant systematic error (or bias, in laboratory vernacular), a constant difference between the new and reference assay regardless of assay result magnitude. A laboratory director may choose to adopt a new assay with systematic error but must modify the published reference interval.

503
Q

The positive predictive value predicts the probability that an individual with a positive assay result ___ the disease or condition. *

Has
Could have
May have
Will have

A

Has

The positive predictive value predicts the probability that an individual with a positive assay result has the disease or condition.

The negative predictive value predicts the probability that an individual with a negative assay result does not have the disease or condition.

504
Q

It describes the total number of events or conditions in a broadly defined population, for instance, the total number of patients with chronic heart disease in the Philippines.

Incidence
Prevalence
False negative
False positive

A

Prevalence

Epidemiologists describe population events using the terms prevalence and incidence.

  1. Prevalence describes the total number of events or conditions in a broadly defined population, for instance, the total number of patients with chronic heart disease in the United States.
  2. Incidence describes the number of events occurring within a randomly selected number of subjects representing a population, over a defined time, for instance, the number of new cases of heart disease per 100,000 U.S. residents per year.

Scientists use incidence, not prevalence, to select laboratory assays for specific applications such as screening or confirmation.